Governance

You might also like

Download as pdf or txt
Download as pdf or txt
You are on page 1of 71

TSPSC Group-I |Naipunyata| A 100-Day Answer Writing Program

Day-71: Governance & Good Governance | Paper-III | Section-III | Unit-I


1. What is Good Governance? Enlist various initiatives of Good Governance at the Centre and the state of
Telangana
Syllabus Paper III: Section III: Good Governance
Keywords What and Enlist
Introduction  According to World Bank (1997), “Good Governance is, among other things, participatory,
transparent and accountable. It is also effective and equitable. And it promotes the rule
of law. Good governance ensures that political, social and economic priorities are based
on broad consensus in society and that the voices of the poorest and the most vulnerable
are heard in decision-making over the allocation of development resources.”
Body The basic objectives of Good Governance include:
 Timely Service Delivery
 Responsible Government
 Transparency & Accountability
 Protection of Rights
e-Governance is one of the best ways to in order to ensure these basic objectives in the New
Digital India
Initiatives:
 The ICT-based governance opened new economic opportunities, brought transparency in
public-private transactions, insights into outsourcing processes and an accountable
administration.
 The application of electronic means in the interaction between government and citizens
(G2C) and government and business (G2B) as well as in internal government operations
(G2G) has simplified and improved democratic government and business aspects of
governance.
 E-governance and digitalisation is changing the way governments are addressing the
problems of the citizens and delivering them.
 Digitalisation will go a long way in making the system accountable and transparent.
Timely Service Delivery:
 Direct Benefit Transfer of Central Sector Schemes like PM KISAN
 Ensuring Minimum Support Price for Kharif and Rabi crops
 Single Window System of procedures and approvals
 The public services law in India is an agreement of contract entered into between the
citizens and the public servants,
 It provides for competent and time bound delivery of services. The public service
guarantee act is also known as Right to Public Services Act in some states
Responsible Government:
 Citizen’s Charter is a document representing an effort to focus on the commitment of the
public organisations towards its clients/citizens.

TSPSC | Group-II | Batch-3 | Admissions in progress | Call 7013495019 or visit www.civiccentre.in


 It was with the motto of ‘Putting People First’ that the Citizen’s Charter initiative came
up as a written declaration by government departments enlisting a set of
commitments, standards of service delivery and redressal and remedial actions in
case of non-compliance with the same.
 Enacting and Constituting Lokpal and Lok Ayukta
 Corporate Social Responsibility (CSR), a governance-centric approach to citizen’s welfare,
is an obligation on the part of the corporates to act in a manner, which will serve the best
interests of the society.
 The concept of CSR is based on the premise that it is not only the State through public
policies, which is responsible for societal development, but the corporates also need
to be responsible for such initiatives.
 Decentralisation and people’s participation through local self-government (73rd and 74th
Amendment acts of 1992)
 Modernizing Police forces and implementing the Model Police Act of 2015
Transparency & Accountability
 Initiatives like e-NAM that acts as an agricultural marketing platform
 DigiLocker: DigiLocker aims at ‘Digital Empowerment’ of citizen by providing access to
authentic digital documents to citizen’s digital document wallet.
 The issued documents in DigiLocker system are deemed to be at par with original
physical documents
 Initiatives like PRAGATI (Proactive Governance and Timely Implementation) that reviews
and monitors various Government Projects across the country.
 CAG Reports to the Parliament
Protection of Rights
 Right to Information has emerged as a prominent concern in the field of public
administration and is regarded as one of the most important social innovations of this
century in the field of accountability and transparency and also public service delivery
 Increase in the number of PIL cases and historic judgements as were in the case of Temple
Entry in different religious places owing to gender equality
Telangana:
Timely Service Delivery
 RTA m-wallet that serves as a platform for vehicle documents and pay penalties
 Industrial approvals within 30 days
 Services like MeeSeva that provides various government services at the last mile
Responsible Government
 Government Order Issue Register (GOIR): The IT, E&C Dept. facilitates all the other
Government Departments to upload their Government Orders (Ms and Rt) through
Online Government Order Issue Register (GOIR) Portal to make them available to citizens.
 eTaal: A web portal for dissemination of e-transaction statistics of National and State level
e-Governance projects including Mission Mode Projects.
 eTaal presents quick analysis of transaction counts in tabular and graphical form to
give quick view of transactions done by various e-Governance projects.
Transparency & Accountability

TSPSC | Group-II | Batch-3 | Admissions in progress | Call 7013495019 or visit www.civiccentre.in


 e-Procurement: The Government e-Procurement Platform is robust, efficient,
transparent, economical, and significantly reduces the time to process the tenders.
 It involves activities related to evaluating and selecting suppliers, such as e-Tenders
and e-Auctions.
 Purchase Control offers everything the department needs to successfully execute e-
procurement.
 Besides, it removes routine tasks from the procurement wing of the department
 e-Office Project: e-File is a workflow-based system that includes the features of existing
manual handling of files in addition to the more efficient electronic mode.
 With this system, the movement of receipts and files become seamless and there is
more transparency in the system since each and every action taken on a file is
recorded electronically.
Protection of Rights
 Increase in the reservation for Scheduled Tribes as per the proportion of their population
in educational opportunities to 10%
 Stringent implementation of Child Labour regulations even in the media industry to
protect the children from harmful effects
Conclusion Balanced/Based on Substantiation/Futuristic based: Good Governance is not the end in itself
it is the means for effective delivery of services and protect the rights of the citizens
2. Examine how financial committees of the parliament ensures accountability of the Executive?
Syllabus Paper III: Section III: Parliamentary Committees
Keyword Examine
Introduction Definition/Significance based:
The financial accountability of the executive to the Legislature is ensured through a system of
committees called the financial committees.
Body  There are three financial committees, namely the Estimates Committee, the Public
Accounts Committee (PAC), and the Committee on Public Undertakings (CoPU) which are
set up under the Rules of Procedure and Conduct of Business in the Lok
Sabha/Vidhansabhas of the States.
Public Accounts Committee:
 The PAC is accorded a pivotal role in ensuring financial accountability.
 The PAC of the Parliament is constituted every year.
 The functions of the Committee include examination of accounts showing the
appropriation of sums granted by the legislature for the expenditure of the Government,
the annual Finance Accounts of the Government and such other accounts laid before the
House as the Committee may think fit.
 In scrutinizing the Appropriation Accounts of the Government and the Report of the CAG
thereon, the Committee has to satisfy:
 that the moneys shown in the accounts as having been disbursed were legally
available for, and applicable to, the service or purpose to which they have been
applied or charged;
 That the expenditure conforms to the authority which governs it;

TSPSC | Group-II | Batch-3 | Admissions in progress | Call 7013495019 or visit www.civiccentre.in


 That every re-appropriation has been made in accordance with the provisions made
in this behalf under unless framed by the competent authority.
 Can mention other duties
 The Functions of the Committee extend however, "beyond the formality of expenditure,
to its wisdom, faithfulness and economy".
 The Committee thus examines cases involving losses, nugatory expenditure and financial
irregularities.
 While scrutinizing the Reports of the CAG on Revenue Receipts, the Committee examines
various aspects of the Government's tax administration, cases involving under-
assessments, tax-evasion, non-levy of duties, misclassifications etc., identifies the
loopholes in taxation laws and procedures and makes recommendations in order to check
leakage of revenue.
 One of the duties of the Committee is to ascertain that money granted by the legislature
has been spent by the Government within the scope of the demand.
 It considers the justification for spending more or less than the amount originally
sanctioned.
 If any money has been spent by the Government on a service in excess of the amount
granted by the House for the purpose, the Committee examines the same with reference
to the facts of each case, as well as the circumstances leading to such an excess and makes
such recommendations as it deems fit.
Estimates Committee:
The term of the committee is one year. The functions of the Estimates Committee are:
 To report what economies, improvements in organisation, efficiency or administrative
reform, consistent with the policy underlying the estimates may be effected
 To suggest alternative policies in order to bring about efficiency and economy in
administration
 To examine whether the money is well laid out within the limits of the policy implied in
the estimates
 To suggest the form in which the estimates shall be presented to Parliament.
The Committee does not exercise its functions in relation to such Public Undertakings as are
allotted to the Committee on Public Undertakings by the Rules of Procedure and Conduct of
Business of Lok Sabha or by the Speaker
Committee on Public Undertakings:
 The Committee on Public Undertaking (CoPU) is constituted every year for the
examination of the working of the public undertakings. In the States also, similar
arrangements exist.
 The functions of the Committee are:
 To examine the Reports and accounts of Public Undertakings;
 To examine, in the context of the autonomy and efficiency of the public undertakings,
whether the affairs of the public undertakings are being managed in accordance with
sound business principles and prudent commercial practices and
 To exercise such other functions as are vested in the PAC and the Estimated
Committee in relation to the public undertakings.
 However, COPU is precluded from examining and investigating

TSPSC | Group-II | Batch-3 | Admissions in progress | Call 7013495019 or visit www.civiccentre.in


 any matters of major Government policy as distinct from business or commercial
functions of the public undertakings;
 Matters of day to day administration and matters for the consideration of which
machinery is established by any special statute under which a particular public
undertaking is established.
Conclusion Balanced:
The financial committees of parliament serve as a system of checks & balances on the
executive's management of public finances, ensuring that it remains accountable to the
people it serves.
3. Social audit proves to be an effective measure in bridging the gap between good governance and social
cognizance. Comment
Syllabus Paper III: Section III: Good Governance
Keyword Comment
Introduction Definition/Report/Context/Significance based:
Social audit is a process of evaluating the effectiveness of the policies and programs of a
government or organization through the participation of citizens or stakeholders. It is a
powerful tool that helps in bridging the gap between good governance and social cognizance
in India.
Body  Social audit is an effective measure in India as it provides a platform for citizens to voice
their opinions and concerns about the policies and programs implemented by the
government.
 This process ensures that the policies and programs are transparent and accountable, and
that they meet the needs of the citizens.
 The Indian government has recognized the importance of social audit and has
implemented it in various sectors.
 The MGNREGA has been successful in providing employment opportunities to
millions of rural households across India, and the social audit process has played a
critical role in ensuring its effectiveness.
 The social audit process for MGNREGA involves the participation of citizens, civil
society organizations, and government officials. The process includes a review of the
records and documents related to the implementation of the scheme, public
hearings, and the submission of reports by the social audit teams.
 The social audit process helps in identifying the gaps and issues in the implementation of
the scheme, which are then addressed by the government.
 The success of the social audit process in India can be attributed to several factors.
 The process ensures that the policies and programs are transparent and accountable.
This transparency helps in building trust between the government and citizens, which
is crucial for good governance.
 The social audit process involves the participation of citizens, which empowers them
and gives them a voice in the decision-making process. This participation helps in
building a sense of ownership and responsibility among citizens towards the policies
and programs.
 The social audit process provides an opportunity for citizens to monitor the
implementation of the policies and programs, which helps in identifying the gaps and

TSPSC | Group-II | Batch-3 | Admissions in progress | Call 7013495019 or visit www.civiccentre.in


issues in the implementation process. This feedback helps in improving the
effectiveness of the policies and programs.
Conclusion Balanced:
Social audit is an effective measure in bridging the gap between good governance and social
cognizance in India. The process helps in ensuring transparency, accountability, and citizen
participation in the decision-making process. The success of the social audit process in India
can be attributed to the participation of citizens, civil society organizations, and government
officials, which helps in identifying the gaps and issues in the implementation of the policies
and programs. Therefore, the Indian government should continue to promote and implement
social audit in various sectors to ensure good governance and social cognizance.
4. Define e-Governance? Explain various types of interactions in E-Governance?
Syllabus Paper III: Section III: E-Governance
Keywords Define and Explain
Introduction Definition/Significance based:
 E-governance is the application of information and communication technologies to
transform the efficiency, effectiveness, transparency and accountability of informational
and transactional exchanges with in government, between government & govt. agencies
of National, State, Municipal and Local levels, citizen & businesses, and to empower
citizens through access & use of information
 World Bank explained the E governance as the use by government agencies of information
technologies (such as Wide Area Networks, the Internet, and mobile computing) that have
the ability to transform relations with citizens, businesses, and other arms of government.
Body Various types of Interactions in e-governance:
Government-to-Citizen (G2C):
 This kind of interface between the government and citizens enable the benefit from a
wide range of public services.
 This facilitates the citizen the availability and accessibility of the government services
anytime, anywhere mode.
 This gives citizen the choice of when to interact with the government 24 hours a day, 7
days a week; from where to interact with the government-service centre, unattended
kiosk or from the comfort of one's home; and how to interact - through internet, fax
telephone, email, face-to-face, etc.
Examples include: Mee Seva, Direct Benefit Transfer etc
Government-to-Business (G2B):
 It is viewed that the business community is provider of good and services.
 The extensive interaction between government and business in case of licencing, permits
procurement and revenue collection helps to cut red tape, saves time, reduces
operational costs, cut unnecessary delays and eliminates redundant data capture.
 It also creates a more transparent business environment when dealing with the
government.
Examples include single window and deemed clearances, land banks etc
Government-to-Government (G2G):

TSPSC | Group-II | Batch-3 | Admissions in progress | Call 7013495019 or visit www.civiccentre.in


 This kind of government to government interaction can be within the sphere of the
government, can be either horizontal i.e. between different government agencies vertical
i.e., between national, provincial and local government agencies.
 It is viewed that this kind of interaction helps to share information in order to avoid
duplication and reduce turn-around time; to increase operational efficiency; and to
facilitate better integration on inter-governmental programmes leading performance
improvements.
PRAGATI, e- Office etc being the examples of it
Government to Employee (G2E):
 Government-to-Employee interaction refers to the delivery of services by the
Government to its employees.
 This type of interface could be primarily related to HR areas. But it helps employees
maintain communication with the government and their own companies.
 This allows Employees learning new technology in one place, the computer.
 Documents can be stored and shared with other colleagues online. This makes employees
to become paperless and makes it easy to send important documents back and forth to
colleagues all over the world.
 It also includes maintaining personal information and records of employees.
Employee Health Insurance, details of Pension and any sort of grievances being some of the
examples
Conclusion Balanced/Significance based:

Hence, e-governance ensures better delivery of government services to citizens, improved


interactions with business and industry, citizen empowerment through access to information,
or more efficient government management. The resulting benefits can be less corruption,
increased transparency, greater convenience, revenue growth, and or cost reductions

5. Critically analyse the Protection of Human Rights (Amendment) Act 2019


Syllabus Paper III: Section III: Constitutional and Statutory bodies
Keyword Critically analyse
Introduction Context/Significance based:
 The Protection of Human Rights (Amendment) Bill, 2019 was introduced in Lok Sabha by
the Minister of Home Affairs, Mr. Amit Shah, on July 8, 2019.
 The Bill amends the Protection of Human Rights Act, 1993. The Act provides for a National
Human Rights Commission (NHRC), State Human Rights Commissions (SHRC), as well as
Human Rights Courts.
Body The Protection of Human Rights (Amendment) Bill, 2019 provides
 That a person who has been a Judge of the Supreme Court is also made eligible to be
appointed as Chairperson of the Commission in addition to the person who has been the
Chief Justice of India;
 To increase the Members of the Commission from two to three of which, one shall be a
woman.

TSPSC | Group-II | Batch-3 | Admissions in progress | Call 7013495019 or visit www.civiccentre.in


 To include Chairperson of the National Commission for Backward Classes, Chairperson of
the National Commission for Protection of Child Rights and the Chief Commissioner for
Persons with Disabilities as deemed Members of the Commission;
 To reduce the term of the Chairperson and Members of the Commission and the State
Commissions from five to three years and shall be eligible for re-appointment;
 To provide that a person who has been a Judge of a High Court is also made eligible to be
appointed as Chairperson of the State Commission in addition to the person who has been
the Chief Justice of the High Court; and,
 To confer upon State Commissions, the functions relating to human rights being
discharged by the Union territories, other than the Union territory of Delhi, which will be
dealt with by the Commission.
Some of the criticisms faced by the amendment include:
Weakening NHRC Credibility:
 Much of NHRCs credibility lies in the fact that it is led by a former Chief Justice of India
(CJI).
 This clause is now proposed to be amended to “broaden” the scope of the NHRC to allow
a former judge of the Supreme Court to be in the running for the top post in the human
rights body.
 This will only allow the appointment committee to handpick the most ‘favourable’ judge
to head the NHRC which is a matter of grave concern.
 If the NHRC needs to be headed by a judge of the Supreme Court, in the interest of the
institution, it is desirable that it is the CJI, hence restricting the government’s influence on
the NHRC and the judiciary.
 In light of GANHRI’s 2011,2016 and 2017 recommendations, and given that quasi-judicial
function is only one out of the ten functions laid out in Section 12 of the PHRA, it is
important to debate if a judicial appointee is best suited to lead the apex human rights
body.
Lack of Diversity:
 The proposed amendment states to increase the number of non-judicial members from
two to three, with one being a woman. GANHRI recommendations on the need for
diversity and pluralism in the NHRC and requirements as per the Paris Principles, seem to
have been answered here with a tokenism.
 In its 25 years, the NHRC has had just three women as members and none as its
chairperson. In the current setup, if the government is serious with its intentions, it should
have proposed that half of its commissioners are women.
 Further, the amendments are silent about the representation of commissioners from
Scheduled Castes, Scheduled Tribes, religious and ethnic minorities communities.
 For a country with over 1.3 billion population, the apex human rights body cannot be left
in the control of five or six commissioners representing the majority socio-economic-
cultural groups.
 The number of commissioners needs to significantly increase to ensure that they can
respond to the large volume of complaints and allow representation from diverse groups.
After all, these are the communities that need an effective NHRC the most.
Shorter terms mean greater government interference:

TSPSC | Group-II | Batch-3 | Admissions in progress | Call 7013495019 or visit www.civiccentre.in


 Shortening the term of commissioners from five to three years is another change that has
not been well thought through.
 A judicial member is appointed after retirement, after 65 years in case of a
chairperson and a judicial member and 62 years in case of another judicial member,
with the cap of 70 years as retirement.
 A three-year term, eligible for reappointment, would practically mean a shorter second
tenure.
 Shorter tenures of commissioners create a larger room for government interference to
replace commissioners who fail to toe the government line.
Becoming Toothless
 The proposed amendments fail to answer concerns about the independence, autonomy,
pluralism and functioning of the NHRC. The transformation from a “toothless” to a
“roaring” commission is misleading.
 For it to bite, there must be a change in the composition of the appointing committee.
This was what was highlighted by the Subcommittee of Accreditation in its various
reports.
 Section 4 of the PHRA provides for a six-member appointment committee guaranteeing
at least two seats to opposition party leaders, one from the Lok Sabha and the other from
the Rajya Sabha. The three guaranteed members are from the ruling coalition – the Prime
Minister, Home Minister and the Speaker of the House. The remaining member – Deputy
Chair of the Rajya Sabha – may or may not belong to the ruling party.
 The composition of the appointment committee would be in the ratio of 4:2 in most cases,
3:3 in rare cases or 5:1 as experienced in the past five years with the absence of a Leader
of Opposition in the Lok Sabha.
 As long as the government’s representation in the appointment committee is in the
majority, no reform or amendment can sufficiently address the concerns with the NHRC.
Need for public debate
 Both the NHRC and government should provide an opportunity for people to submit
comments. The NHRC has not commented on the proposed amendments. NHRC’s
recommendations on the PHRA amendments are not in the public domain.
 Concerns on the proposed amendments have been raised by the opposition in the Lok
Sabha. It is imperative that the NHRC submits publicly its position on these proposed
amendments.
 As this will now be tabled in the Rajya Sabha, it should permit time for greater public
debate on the amendments.
Conclusion Futuristic based:
The Bill can be referred to a Standing Committee of the Parliament to obtain wider views on
the subject and be closely studied in the light of the SCA recommendations as well as the
communication of the UN High Commissioner for Human Rights to the Government of India
last year because compromising with the NHRC means allowing over 169 similar institutions
at the national and state level to be totally decimated in this country.

TSPSC | Group-II | Batch-3 | Admissions in progress | Call 7013495019 or visit www.civiccentre.in


TSPSC Group-I |Naipunyata| A 100-Day Answer Writing Program
Day-72: Governance at State and District Levels | Paper-III | Section-III | Unit-II
1. Assess the role of the Finance Commission in the devolution of powers to the local bodies.
Syllabus Paper-III | Section-III | Unit-I: Finance commission
Keyword Assess
Introduction Article based: The Finance Commission is a Constitutionally mandated body that is at the
centre of fiscal federalism. Set up under Article 280 of the Constitution, its core responsibility
is to evaluate the state of finances of the Union and State Governments, recommend the
sharing of taxes between them, and lay down the principles determining the distribution of
these taxes among States.
 Article 281 deals with the recommendations of Finance Commission
Body  The first Finance Commission was set up in 1951 and there have been fifteen so far. While
the Constitution of India requires a Finance Commission (FC) to be set up every five years,
the 15th FC’s mandate was extended by a year till 2025-26, breaking the cycle.
 The Finance Commission is required to make recommendations to the President of India
on the following matters
 The principles that should govern the grants-in-aid to the states by the Centre (i.e.,
out of the Consolidated Fund of India).
 The measures needed to augment the Consolidated Fund of a State to supplement
the resources of the panchayats and the municipalities in the state based on the
recommendations made by the state finance commission.
 The trend of devolution to local bodies was a rarity before 14th Finance Commission.
14th Finance Commission (FC) and its recommendations to local bodies
 The 14th Finance Commission (FFC) was constituted by the orders of President on 2nd
January, 2013 and submitted its report on 15th December, 2014
 14th FC has recommended distribution of grants to States for local bodies using 2011
population data with weight of 90% and area with weight of 10%.
 14th FC has recommended grants in two parts; a basic grant, and a performance grant, for
duly constituted Gram Panchayats and Municipalities.
 The ratio of basic to performance grant is 90:10 with respect to Panchayats and 80:20
with respect to Municipalities.
 14th FC has recommended a total grant of ₹2, 87, 436 crore for five-year period from
1.4.2015 to 31.3.2020. Grant recommended to
 Panchayats is ₹ 2,00,292.20 crores
 Municipalities are ₹ 87,143.80 crores.
 The transfers in the year 2015-16 will be ₹ 29,988 crores.
th
15 Finance Commission and its recommendations to local bodies- chairman is Mr. N. K.
Singh
The final report with recommendations for the 2021-26 period was tabled in Parliament on
February 1, 2021. Key recommendations to local bodies include:
 Grants to local bodies (other than health grants) will be distributed among states based
on population and area, with 90% and 10% weightage, respectively.
 The total grants to local bodies will be ₹ 4.36 lakh crore (a portion of grants to be
performance-linked) including:
 ₹ 2.4 lakh crore for rural local bodies,
 ₹ 1.2 lakh crore for urban local bodies

TSPSC | Group-II | Batch-3 | Admissions in progress | Call 7013495019 or visit www.civiccentre.in


 ₹ 70,051 crore for health grants through local governments.
 The grants to local bodies will be made available to all three tiers of Panchayat- village,
block, and district.
 The health grants will be provided for:
 conversion of rural sub-centres and primary healthcare centres (PHCs) to health and
wellness centres (HWCs)
 support for diagnostic infrastructure for primary healthcare activities
 support for urban HWCs, sub-centres, PHCs, and public health units at the block
level.
 The Commission has prescribed certain conditions for availing these grants (except health
grants). The entry-level criteria include:
 Fixation of minimum floor rates for property taxes by states and improvement in the
collection of property taxes (an additional requirement after 2021-22 for urban
bodies).
 No grants will be released to local bodies of a state after March 2024 if the state does
not constitute the State Finance Commission and act upon its recommendations by
then.
Grants to Telangana local bodies through Fifteenth Finance Commission
 Rural local bodies- ₹7,201cr, Urban local bodies- ₹3,682cr
Recent Context
Process to set up Sixteenth Finance Commission set to kick off soon
 Finance Ministry likely to notify the terms of references for the constitutional body,
tasked with recommending the revenue sharing formula between the Centre and States
and their distribution among States, towards the latter half of this year
Conclusion Substantiation based: The Finance Commission's allocation of resources to local bodies is
critical for the provision of essential services, the empowerment of local governance, and the
promotion of accountability, transparency, and participatory democracy.
2. Discuss the devolution of powers to rural local governments under 73rd Constitutional Amendment 1992 to
the Indian Constitution
Syllabus Paper-III | Section-III | Unit-I: 73rd CAA and powers of Panchayat Raj system
Keywords Discuss
Introduction Definition based: The term Panchayati Raj in India signifies the system of rural local self-
government. Significant committees like Balwant Rai Mehta committee, Ashok Mehta
Committee recommended for setting up local bodies. It was constitutionalized through the
73rd Constitutional Amendment Act of 1992.
 This act has added a new Part-IX to the Constitution of India. This part is entitled as ‘The
Panchayats’ and consists of provisions from Articles 243 to 243 O.
Body Key objectives of 73rd Constitutional Amendment 1992
 The act provides for the reservation of seats for scheduled castes and scheduled tribes in
every panchayat
 The act provides for the reservation of not less than one-third of the total number of seats
for women (including the number of seats reserved for women belonging to the SCs and
STs).
 The act provides for a five-year term of office to the panchayat at every level. However,
it can be dissolved before the completion of its term.
Powers

TSPSC | Group-II | Batch-3 | Admissions in progress | Call 7013495019 or visit www.civiccentre.in


 The preparation of plans for economic development and social justice
 The implementation of schemes for economic development and social justice as may be
entrusted to them, including those in relation to the 29 matters listed in the Eleventh
Schedule.
 Finances: The state legislature may authorise a panchayat to
 Levy, collect appropriate taxes, duties, tolls and fees
 Assign to a panchayat taxes, duties, tolls and fees levied and collected by the state
government
 Provide for making grants-in-aid to the panchayats from the consolidated fund of the
state
 Provide for constitution of funds for crediting all monies of the panchayats.
 Implementation of Government Schemes: The Gram Panchayats are responsible for
implementing various government schemes related to poverty alleviation, employment
generation, and social welfare.
 Management of Local Markets: Gram Panchayats have the power to regulate and manage
local markets, fairs, and other trading activities within their jurisdiction
Recent context: Rural Development and Panchayati Raj Minister Giriraj Singh on Thursday
launched the Mission Antyodaya Survey 2022-23 to assess the outcome of various
development schemes in rural areas.
Conclusion Futuristic based: In conclusion, the devolution of powers to Panchayati Raj Institutions has
already made a significant impact on local governance in India. However, there is always
scope for further devolution of powers to strengthen the grassroots governance system and
ensure greater participation of local communities in decision-making.
3. Explain the role of District Collector in the administrative hierarchy of the executive. Do you think it is still
under colonial influence?
Syllabus Paper-III | Section-III | Unit-I:: Role of District collector
Keywords Explain
Introduction Importance/Significance based: District administration is the total functioning of government
in a district, the total and complex organisation of the management of public affairs at work,
dynamic and not static, in the territory of a geographically demarcated district. District
administration provides the principal points of contact between the citizens and processes of
government, it is truly the cutting edge of the total of public administration; and this is what
constitutes its vital significance in the nation’s government.
Body A District Collector is recruited by Union Public Service Commission under Article-315 of the
Constitution that constituted All India Services under Article-312 of the Constitution of India
District Collector
 The district officer or the collector is the head of district administration in India.
 The District Collector is responsible for overseeing the administration of the district and
ensuring the implementation of government policies and programs.
 The Indian district collector derives their power from the Constitution of India and various
statutes and rules enacted by the central and state governments.
Functions of Collector
 Maintaining law and order in the district
 Collecting revenue and taxes
 Implementing government policies and schemes
 Conducting elections in the district

TSPSC | Group-II | Batch-3 | Admissions in progress | Call 7013495019 or visit www.civiccentre.in


 Supervising the work of other government departments in the district
 Dealing with natural disasters and emergencies
 Hearing and resolving public grievances.
Change in the role of collector from British period to till now
 During the British period, the district collector was primarily responsible for collecting
revenue and maintaining law and order in the district.
 They were considered to be the chief revenue officer of the district and had extensive
powers to collect taxes, settle disputes related to land ownership, and oversee the
functioning of the local administration.
 After India gained independence in 1947, the role of the district collector evolved to
include a wider range of responsibilities.
 The district collector became the head of the district administration and was
responsible for implementing various development schemes and programs in the
district.
 They were also responsible for maintaining law and order, conducting elections, and
overseeing the functioning of various government departments in the district.
 In recent years, the role of the district collector has become even more diverse and
complex.
 District collectors are now responsible for a wide range of tasks, including disaster
management, public health, education, and social welfare.
 They are also expected to work closely with local communities and other stakeholders to
ensure that development programs are implemented effectively and that the needs of
the people are met.
Recent context: Collector warns of legal action against corporation of Cochin for its failure to
dispose of the waste generated in the city.
Conclusion Balanced based:
In conclusion, while the role of the district collector in India may have its roots in the British
colonial period, the collector's responsibilities have evolved significantly over the years.
Today, the collector is an important administrative officer who plays a crucial role in the
development and governance of the district. The collector's role is not solely influenced by
colonialism but is an integral part of India's administrative system.

4. Cooperatives help in achieving the economic, social and cultural progress in the societies. Substantiate
Syllabus Paper-III | Section-III | Unit-I: Cooperatives
Keyword Substantiate
Introduction Definition based: A co-operative is an autonomous group of people who have come together
voluntarily to achieve their common economic, social, and cultural needs and objectives
through a democratically governed and collectively owned business. They are founded on the
principles of self-reliance, accountability, democracy, equality, equity, and solidarity. Co-
operative members believe in the ethical qualities of honesty, openness, social responsibility,
and care for others, as their founders did.
Body Constitutional provisions related to Cooperative societies
 In this context, 97th CAA (2011) made the following three changes in the constitution:
 It made the right to form co-operative societies a fundamental right (Article 19).
 It included a new Directive Principle of State Policy on promotion of co-operative
societies (Article 43-B).

TSPSC | Group-II | Batch-3 | Admissions in progress | Call 7013495019 or visit www.civiccentre.in


 It added a new Part IX-B in the Constitution which is entitled “The Co-operative
Societies” (Articles 243-ZH to 243-ZT)
Role of Cooperatives in Economic progress
 The Cooperative sector plays a pivotal role in strengthening the rural economy.
 The cooperative societies engaged, particularly, in the rural based sector such as
agriculture, fishery, agro-processing, dairy are providing credit, agricultural inputs and
marketing for milk, fish, vegetable, fruits, flower, medicinal plants, forest products, honey
& resham etc.
 While direct employment is provided by the concerned Cooperative Societies, the
strengthening of the entire ecosystem of the Cooperative sector generates huge
opportunities of indirect employment in the rural sector.
 Government has taken many initiatives that includes
 Preparation of a new National Cooperation Policy
 Consolidating a National Cooperative Database
 Preparing Sahakar se Samriddhi scheme while computerising the existing Primary
Agriculture Cooperative Societies (PACS)

Role of Cooperatives in Social progress


 Cooperatives contribute to the social progress of a society by promoting social inclusion
and community development.
 Cooperatives are often formed by marginalized groups, such as women, farmers, and low-
income households, to address their economic and social needs.
 By working together, members can build social capital, strengthen their bargaining
power, and promote their collective interests. This, in turn, helps to build stronger and
more cohesive communities.
 Kudumbashree: Kudumbashree is a women's cooperative in Kerala that was formed
in 1998 to empower women and promote rural development.
 The Self-Employed Women's Association (SEWA) is a women's cooperative that was
formed in Gujarat in 1972 to empower women workers.
Role of cooperatives in cultural progress

TSPSC | Group-II | Batch-3 | Admissions in progress | Call 7013495019 or visit www.civiccentre.in


 Cooperatives such as the Indian Coffee House and the Seva Cafe have provided a space
for artists, writers, and intellectuals to gather and exchange ideas.
 Cooperatives such as Amul and Kudumbashree have helped to preserve traditional
farming practices and handicrafts by providing training, finance, and marketing support
to farmers and artisans.
 Cooperatives such as the National Handicapped Finance and Development Corporation
and the National Scheduled Castes Finance and Development Corporation have provided
finance and training support to disabled and Dalit entrepreneurs, helping them to
promote their cultural traditions and contribute to the overall development of India.
Challenges cooperatives are facing
 Government and legislative control of cooperatives increased over the years,
 There were increasing reports of mismanagement and corruption.
Recent context:
 To plug the “loopholes” in the MSCS Act, the Centre introduced a Bill seeking to amend
the 2002 law for more “transparency” and increase the “ease of doing business”.
 RTI Act will not apply to cooperative societies, says Madras High Court
Conclusion Balanced/Futuristic based: Cooperatives are formed to meet the common economic, social,
and cultural needs of their members and can contribute to the overall development of
communities. By promoting entrepreneurship, creating job opportunities, promoting social
inclusion, and preserving local culture and traditions, cooperatives can help to build stronger,
more equitable, and more resilient societies.
5. Highlight the role of National Commission for SCs in ensuring the economic progress of the scheduled
castes.
Syllabus Paper V: Section I: National Commission for Scheduled Caste
Keyword Highlight
Introduction Constitutional Article based:
 The National Commission for Scheduled Tribes (NCST) has been created by amending
Article 338 and inserting a new Article 338A in the Constitution through the Constitution
(89th Amendment) Act, 2003.
 By this amendment, the erstwhile National Commission for Scheduled Castes and
Scheduled Tribes was replaced by two separate Commissions namely- (i) National
Commission for Scheduled Castes (NCSC), and (ii) National Commission for Scheduled
Tribes in 2004
Body National Commission for Scheduled Castes (NCSC):
 NCSC is a constitutional body that works to safeguard the interests of the scheduled
castes (SC) in India.
 It consists of a chairperson, a vice-chairperson and three other members.
 They are appointed by the President by warrant under his hand and seal.
 Their conditions of service and tenure of office are also determined by the President.
Nature and Functions:
 Monitoring and investigating all issues concerning the safeguards provided for the SCs
under the constitution.
 Enquiring into complaints relating to the deprivation of the rights and safeguards of the
SCs.

TSPSC | Group-II | Batch-3 | Admissions in progress | Call 7013495019 or visit www.civiccentre.in


 Taking part in and advising the central or state governments with respect to the planning
of socio-economic development of the SCs. Regular reporting to the President of the
country on the implementation of these safeguards.
 Recommending steps to be taken to further the socio-economic development and other
welfare activities of the SCS. Any other function with respect to the welfare, protection,
development and advancement of the SC community.
 The Commission is also required to discharge similar functions with regard to the Anglo-
Indian Community as it does with respect to the SCs.
Schemes by NCSC for the economic progress of Scheduled Castes
 Entrepreneurship Development Scheme for SCs: The scheme provides loans at low
interest rates, and also provides training and mentoring support to SC entrepreneurs.
 Dr. Ambedkar Pre-Matric and Post-Matric Scholarship Scheme: This scheme aims to
provide financial assistance to SC students for pursuing education at the pre-matric and
post-matric levels.
 Scheme for Rehabilitation of Manual Scavengers: This scheme aims to rehabilitate
manual scavengers and their dependents by providing them with alternative livelihood
options, such as skill training and entrepreneurship development.
 National Overseas Scholarship for SCs: This scheme provides financial assistance to SC
students for pursuing higher education abroad
Thus, National Commission for Scheduled Castes plays a major role in safeguarding the rights
Conclusion Futuristic based:
Overall, the NCSC has an important role to play in promoting the rights and welfare of
Scheduled Castes in India. By taking proactive measures like Conduct research and analysis of
SC issues, Advocate for legislative changes etc which can help to create a more equitable and
just society for all.

TSPSC | Group-II | Batch-3 | Admissions in progress | Call 7013495019 or visit www.civiccentre.in


Figure 1: Telangana Revenue Dept. Organisational Chart as per
https://ccla.telangana.gov.in/AboutUs.do

TSPSC | Group-II | Batch-3 | Admissions in progress | Call 7013495019 or visit www.civiccentre.in


TSPSC Group-I |Naipunyata| A 100-Day Answer Writing Program
Day-73: Programs, Agencies and Institutions | Paper-III | Section-III | Unit-III
1. How is poverty measured in India? Evaluate how far India is successful in the poverty alleviation after its
independence.
Syllabus Paper III: Section III: Poverty Alleviation
Keyword How, Evaluate
Introduction World Bank defines poverty as deprivation in well-being comprising many dimensions. It includes
low incomes and the inability to acquire the basic goods and services necessary for survival with
dignity.
Body How Poverty was measured in India?
Historically
 Dadabhai Naoroji did one of the earliest estimations of poverty in his book, ‘Poverty and the
Un-British Rule in India’.
 He formulated a poverty line ranging from Rs 16 to Rs 35 per capita per year, based on
1867-68 prices.
 Next, in 1938, the National Planning Committee (NPC) estimated a poverty line ranging from
Rs 15 to Rs 20 per capita per month.
 In 1944, the authors of the ‘Bombay Plan’ (Thakurdas et al 1944) suggested a poverty line of
Rs 75 per capita per year.
Post-Independence
 VM Dandekar and N Rath made the first systematic assessment of poverty in India in 1971,
based on National Sample Survey (NSS) data from 1960-61.
 They argued that the poverty line must be derived from the expenditure that was
adequate to provide 2250 calories per day in both rural and urban areas.
Alagh Committee (1979)
 In 1979, a task force constituted by the Planning Commission for the purpose of poverty
estimation, chaired by YK Alagh, constructed a poverty line for rural (₹49.1 per capita per
month and urban areas (₹56.7 per capita per month) based on minimum consumption
expenditure.
Lakhadwala Expert Group (1993):
 In 1989, The Planning Commission constituted the Lakdawala Expert Group to "look into the
methodology for estimation of poverty and to re-define the poverty line, if necessary" that
retained previous Alagh committee recommendations based on nutritional requirements.
Tendulkar Committee(2009):
 It recommended incorporation of private expenditure on health and education while
estimating poverty.
 It validated the poverty lines by checking the adequacy of actual private consumption
expenditure per capita near the poverty line on food, education and health by comparing
them with normative expenditures consistent with nutritional, educational and health
outcomes respectively.
 Instead of monthly household consumption, consumption expenditure was broken up into
per person per day consumption, resulting in the figure of ₹ 32 and ₹ 26 a day for urban and
rural areas.
 The national poverty line for 2011-12 was estimated at ₹ 816 per capita per month for rural
areas and ₹ 1,000 per capita per month for urban areas.
Rangarajan Committee(2014):

TSPSC | Group-II | Batch-3 | Admissions in progress | Call 7013495019 or visit www.civiccentre.in


 It reverted to the practice of having separate all-India rural and urban poverty line baskets
and deriving state-level rural and urban estimates from these.
 It recommended separate consumption baskets for rural and urban areas which include food
items that ensure recommended calorie, protein & fat intake and non-food items like clothing,
education, health, housing and transport.
 This committee raised the daily per capita expenditure to ₹ 47 for urban and ₹ 32 for rural
from ₹ 32 and ₹ 26 respectively at 2011-12 prices. Monthly per capita consumption
expenditure of ₹ 972 in rural areas and ₹ 1407 in urban areas is recommended as the poverty
line at the all India level.
Multi-dimensional Poverty Index(MPI):
 Global MPI was launched in 2010 by the United Nations Development Program (UNDP) and
the Oxford Poverty and Human Development Initiative (OPHI),
 Global MPI is a measure of multidimensional poverty covering more than 100 developing
countries.
 A person is multi-dimensionally poor if she/he is deprived in one third or more (means
33% or more) of the weighted ten indicators.
 The Global MPI ranges from 0 to 1, higher values implying higher poverty.
 It is the product of the incidence of poverty (proportion of poor people) and the intensity
of poverty (average deprivation score of poor people).
 The 2021 National MPI Project in India is aimed at deconstructing the Global MPI and creating
a globally aligned and yet customised India MPI
 It draws up comprehensive Reform Action Plans with the larger goal of improving India’s
position in the Global MPI rankings.
 NITI Aayog is the nodal Ministry for the Multidimensional Poverty Index (MPI) ranking States
and Union Territories based on their performance
 The Baseline Report of MPI is based on the National Family Health Survey (NFHS) 4 taken up
during 2015-16.
 NFHS is conducted by the International Institute for Population Sciences (IIPS) under the
Ministry of Health and Family Welfare, Government of India
 The MPI identifies 25.01 per cent of the population as multi dimensionally poor.
 The progress of the country with respect to this baseline will be measured using the NFHS-
5 data collected in 2019-20.
 As per the MPI Report,
 More than 400 million people were lifted out of poverty in India between 2005-06 to
2019-21

TSPSC | Group-II | Batch-3 | Admissions in progress | Call 7013495019 or visit www.civiccentre.in


 The percentage of population as multi dimensionally poor in rural and urban areas is
32.75 per cent and 8.81 per cent.

Conclusion Balanced/Futuristic based:

TSPSC | Group-II | Batch-3 | Admissions in progress | Call 7013495019 or visit www.civiccentre.in


The evolution of poverty lines suggest that India has made significant progress in poverty
alleviation since its independence. However, poverty remains a significant challenge in the country
and there is a need for continued efforts to address this issue and to achieve significant success in
achieving Sustainable Development Goals too.
2. Analyse how far Telangana is successful in the implementation of the welfare programs for women
Syllabus Paper III: Section III: Women Empowerment
Keyword Analyse
Introduction Definition/Report/Context/Significance based:
A Special Bulletin on Maternal Mortality in India 2018-20 released by the Ministry of Home Affairs
in 2022 revealed that the Maternal Mortality Rate in Telangana had come down to 43 in 2020 from
92 in 2014. This shows that significant concentration is shown on the welfare of women in Bangaru
Telangana.
Body Some of the significant initiatives implemented by Telangana government for the welfare of
women include:
Health
Aarogya Lakshmi:
 To alleviate women’s health and welfare, the Government has enhanced the rates by
providing an additional allocation of ₹ 14 per beneficiary per day over and above the norms
mandated by the Government of India.
 Additionally, the Government has also increased the number of days the beneficiaries receive
milk and eggs from 25 to 30.
 In 2022-23 the scheme benefited over 19.07 lakh pregnant, lactating women and children in
the age group of 7 months to 6 years (covered under Balamrutham and Supplementary
Nutrition Programme), the expenditure incurred under Arogya Lakshmi Scheme is Rs.131.35
crore (Till December 2022).
KCR Kit:
 The “KCR Kit” initiative, which aims to lower maternal and infant mortality rates by
encouraging institutional births, was introduced in June 2017.
 Under the scheme, post-delivery mothers receive a kit containing 15 utility items and financial
assistance of ₹12,000 (₹13,000 for a girl child) to compensate for the loss of wages by the
women during the pregnancy and postnatal period.
 Since the scheme’s inception, 13.91 lakh beneficiaries have received a Direct Benefit Transfer
of ₹ 1,261.67 crore, along with KCR Kits.
 The impact of the scheme can be seen in improved institutional deliveries in the state from
30.5% to 61.0%. Telangana’s MMR dropped from 92 in 2014 to 43 in 2020, i.e., a 53% decline,
and the state now ranks third in the country.
KCR Nutrition Kit:
 The state is committed to further decreasing anaemia by introducing another flagship
program called “KCR Nutrition Kit’’ in high anaemia prevalence districts.
Standard of Living
Land Distribution to Dalits:
 The Government provides 3 acres of agricultural land to landless SC women, along with the
provision for creation of irrigation facilities, land development and other agricultural inputs
for their sustained livelihood.
 Under this scheme an extent of 17,096.31 acres of land have been purchased and distributed
to 6,995 beneficiaries at a cost of ₹ 768.94 crore from 2014-15 to 2022-23 (till January 2023).
Self-Help Groups:
 The government further encourages the SHG movement through the SERP (Society for
Elimination of Rural Poverty) by building and promoting women’s SHGs and their associations.

TSPSC | Group-II | Batch-3 | Admissions in progress | Call 7013495019 or visit www.civiccentre.in


 Nearly 46.08 lakh women were organized into 4.30 lakh SHGs across the state. In 2022-23
newly 269 SHGs were formed until December 2022.
 Telangana has facilitated the bank linkages to all the SHGs in the State. The amount disbursed
to the SHGs has been tripled from ₹3,738.67 crore in 2014-15 to ₹12,684.59 crore by 2022-23
(until January 2023) from the State formation.
Welfare
Kalyana Lakshmi:
 The Government has launched a scheme called ’Kalyana Lakshmi/Shaadi Mubarak’ to curtail
the burden of girl marriage by providing one-time grant of Rs.1,00,116 to unmarried girls
(above 18 years of age) from SC, ST, BC and Minority families (with combined income of both
parents not exceeding Rs.2,00,000 per annum) at the time of their marriage.
 From mid-2019, The Government is also providing financial assistance of ₹ 1,25,145 to
unmarried disabled girls irrespective of their caste and religion.
 Up to January 2023, a total of 2.4 lakh SC, 1.5 lakh ST, 5.9 lakh BC, 2.4 lakh Minority families
benefited from the scheme and an amount of ₹ 10,558.79 crore has been disbursed.
 This initiative not only curbed child marriages to a significant extent but also greatly relieved
the financial burden related to marriages.
Mobile Retail Fish Outlets
 A new welfare scheme for unemployed women has been launched in GHMC limits in 2020
called “Mobile retail Fish Outlets” which provides fresh fish and fish curries at the door step
through which 150 unemployed women have been benefited.
Security, Health and Environment (SHE) Teams:
 SHE Teams is a division of Telangana Police established in 2014 which aims to deal with
offences against women like eve-teasing, stalking, and harassment in public or at workplaces,
educational institutions, residential areas or social media.
 At present, there are 331 SHE teams working in the state. Between October 2014-22, a total
of 42,788 complaints were received under the initiative, leading to the arrest of over 26,106
perpetrators
Conclusion Balanced/Futuristic based:
Telangana has been successful in implementing various welfare programs for women. These
programs have helped women in the state to improve their socio-economic status and lead a
better life. However, there are still challenges that need to be addressed, such as ensuring the
effective implementation of these programs at the grassroots level, providing equal opportunities
for women in education and employment, and tackling issues such as domestic violence and
gender discrimination. Nevertheless, Telangana's initiatives towards women's welfare are a
positive step towards gender equality and women empowerment in the state.
3. Explain the concept of social justice with suitable examples from Telangana.
Syllabus Paper III: Section III: People centred participatory development
Keywords Explain
Introduction Definition/Report/Context/Significance based:
Social justice refers to a fair and equitable division of resources, opportunities, and privileges in
society. Social justice is a fundamental principle of a just society, which ensures that every
individual is treated equally, regardless of their socio-economic status, gender, race, religion, or
any other factor. To ensure such an egalitarian society, historical wrongs should be corrected for
which affirmative actions for marginalised sections like SCs, STs, Minorities and Women are taking
place.
Body Some of the examples that depict the achievement of Social Justice in Telangana include:
Dalit Bandhu: Ensures Economic Justice
 A major initiative for the economic upliftment of the Scheduled Caste people.

TSPSC | Group-II | Batch-3 | Admissions in progress | Call 7013495019 or visit www.civiccentre.in


 A complete grant without any bank dependencies to establish a suitable income-generating
source is provided with financial assistance of ₹ 10 lakh for each beneficiary.
 The scheme was launched on August 4, 2021, in the village of Vasalamarri, in the Alair
Assembly Constituency (AC) of the Yadadri-Bhuvanagiri district.
 A provision of ₹ 4,441 crore has been prepared for the year 2021-22, of which ₹ 4150 crore
has been distributed to the districts to cover approximately 38,323 SC–identified households.
 The execution of this project will cost ₹ 17,700 crore for the year 2022–23, with a target of
1500 beneficiaries per AC.
SC, ST Special Development Fund: Ensures Economic Justice
 The Government enacted the “Scheduled Castes and Scheduled Tribes Special Development
Fund (Planning, Allocation and Utilization of Financial Resources) Act in 2017” to secure funds
for the implementation of programmes that ensure accelerated development of SCs and STs
with an focus on economic, educational, and human development along with ensuring their
security and social dignity, and promoting equity among them.
 There is provision for non-lapsing of these funds on a year-by-year basis. Telangana is the first
State to have such a provision.
 Since the creation of the fund (until January 2023), specific funds of approximately ₹ 70,307
crore and ₹ 37,777 crore have been used exclusively for the benefit of SC and ST.
Reservations for Minorities: Ensures social and economic Justice
 Recently the government has increased the Reservation of ST in Educational Institutions and
State Government Services from 6% to 10% on 30th September 2022.
 There are other schemes and Policies like Giri Vikasam that promotes nutritional security,
Scholarships and Residential schools and Colleges that provides financial assistance to
education are some of the initiatives, which ensure Social Justice.
Two-BHK Housing Programme: Ensures Social Justice (Shelter)
 The Government announced a 2-BHK housing program in October 2015, with the aim of
providing cheap and quality housing to those in need.
 A total of 2,92,057 homes were approved under this initiative between 2016 and 2022. Of
these, the construction of 1,36,039 houses was completed by December 2022, while the
construction of the remaining houses is in various stages of completion.
 Around ₹ 11,635.14 crore has been spent on the scheme since its inception to December 2022.
Public Distribution System: Ensures Social Justice (Food):
 To the priority households the essential commodities and food grains have been distributed
by the Government at subsidised prices as per their eligibility.
 The Public Distribution system with a network of 17,216 Fair Price Shops has been proactively
been implemented by Government
 Also Economic Support Schemes like Sheep Rearing Development Scheme Free Electricity
Supply to SC, ST and BC Communities ensures economic justice.
Ensuring cultural upliftment of the Tribal Societies:
 Constructed a tribal museum at Jodeghat, Medaram, Bhadrachalam and Mannanur. New
Ramji Gond Memorial Tribal Freedom Fighters museum is coming up in Abids, Hyderabad.
 The Kumuram Bheem Adivasi Bhavan and Sevalal Banjara Bhavan constructed with ₹ 22.00
Crore each in the heart of the city in Banjara Hills inaugurated by the Hon’ble Chief Minister
on September 17, 2022.
 Government provides support to festivals like Sammakka-Sarakka Jathara, Bathukamma,
Ramzan and Christmas feasts.
Schemes related to Women(previous question) that promotes welfare of the women which also
comes under social justice

TSPSC | Group-II | Batch-3 | Admissions in progress | Call 7013495019 or visit www.civiccentre.in


Conclusion Balanced/Based on Substantiation/Futuristic based:
Social justice is critical for an egalitarian society because it promotes equality, fairness, and
inclusion. It ensures that everyone has the same opportunities to succeed and contribute to
society, which is essential for creating a more just and democratic world.
4. According to the Global Nutrition Report of 2021, India is not on track to achieve the 2030 targets for eradicating
hunger, food insecurity and malnutrition. In this context, discuss some of the initiatives of the Centre and the
Telangana state in achieving these goals.
Syllabus Paper III: Section III: Food Security
Keyword Discuss
Introduction Definition/Report/Context/Significance based:
Sustainable Development Goal 2 is about creating a world free of hunger by 2030. According to
the 2021 Global Nutrition Report, India is not on track on achieving five of the six global maternal,
new born and young child nutrition targets to address — stunting, wasting, anaemia, low birth
weight and childhood obesity. The factors include less concentration on nutritional security and
the life styles
Body Some of the initiatives that promotes nutritional security eradicating hunger, food insecurity and
mal nutrition include:
POSHAN Abhiyaan:
 National Nutrition Mission, also known as POSHAN (The Prime Minister’s Overarching Scheme
for Holistic Nutrition) Abhiyaan is Government of India’s flagship programme to improve
nutritional results for children, pregnant women and lactating mothers.
 POSHAN Abhiyaan has the aim to build a people’s movement (Jan Andolan) around
malnutrition and intends to significantly reduce malnutrition in the following three years.
 The Abhiyaan targets to reduce stunting, undernutrition, anaemia (among young children,
women and adolescent girls) and reduce low birth weight by 2%, 2%, 3% and 2% per
annum respectively.
 The target of the mission is to bring down stunting among children in the age group 0-6
years from 38.4% to 25% by 2022.
National Food Security Mission:
 National Food Security Mission (NFSM) was launched in the year 2007-08 with the aim to
increase the production of rice, wheat and pulses through
 Expansion of area and productivity enhancement,
 Restoring soil fertility and productivity,
 Generating employment opportunities and
 Enhancing farm level economy.
 From 2014-15 coarse cereals were also included in the Mission under NFSM. There are various
interventions covered under NFSM, like Demonstrations of cluster on improved package of
practices, Demonstrations on cropping system, Distribution of Seed having high yielding
varieties etc.
National Food Security Act 2013
 The enactment of the National Food Security Act, (NFSA) 2013 on July 5, 2013 marks a
paradigm shift in the approach to food security from welfare to rights based approach.
 The Act legally entitles up to 75% of the rural population and 50% of the urban population
to receive subsidized food grains under Targeted Public Distribution System.
 About two thirds of the population therefore is covered under the Act to receive highly
subsidized food grains.
 As a step towards women empowerment, the eldest woman of the household of age 18
years or above is mandated to be the head of the household for the purpose of issuing of
ration cards under the Act.

TSPSC | Group-II | Batch-3 | Admissions in progress | Call 7013495019 or visit www.civiccentre.in


 The act provides a legal backing to the mid-day meal program for primary and upper primary
schools children
 It was renamed PM Poshan Shakti Nirman or PM Poshan in 2021 to increase enrolment in
schools and provide food security to children
Zero Hunger Programme:
 The Zero Hunger Programme is an ambitious government scheme with the objective of
reducing hunger across the nation. The Zero Hunger Programme in India was started in 2017
to improve agriculture, health and nutrition.
 The Zero Hunger Programme aims at:
 Decreasing child stunting for children 2 years and younger
 Making sure access to food all year round
 Creating stable food systems
 Increasing small farmer productivity and income
 Eliminating food wastage
 The program also emphasizes on developing farm equipments, revamping the farming system,
creating genetic gardens for biofortified plants and beginning zero hunger training.
Eat Right India Movement:
 The Eat Right India movement is a massive effort to transform the nation’s food system into
safer and healthier eating habits.
 This campaign holds importance as it is not just aims at developing good food habits for the
people of the nation but also promotes food that is good for the planet. It adopts a perfect
mixture of regulatory, capacity building, collaborative and empowerment approach to make
sure that both the parameters are followed.
 The movement also considers a coordinative or ‘whole of the government’ approach since the
movement unites food-related mandates of the agriculture, health, environment and other
ministries.
Telangana:
Some of the prominent nutritional schemes implemented by Telangana include:
 The state is committed to further decreasing anaemia by introducing another flagship
program called “KCR Nutrition Kit’’ in high anaemia prevalence districts. The contents of the
kit are highly nutritious with proteins, micro and macronutrients and the supplements
intended to address anaemia and improve body mass index.
 Balamrutam: Balamrutham is the weaning food introduced under ICDS to provide improved
supplementary nutrition to children between 7 months to 3 years.
 Balamrutham is distributed in packets of 2.5 Kg per child per month.
 It is distributed on first day of every month on Nutrition Health Day-1 to mothers of 7
months - 3 years children as Take Home Ration
 Giri Poshana: To extend nutrition food for tribal people, International Crops Research Institute
for Semi Arid Tropics (ICRISAT) and Department of Tribal Welfare, Telangana, have jointly
launched Giri Poshana - a tribal nutrition project in January 2019.
 The aim of the project is to provide nutritional support to tribal women (pregnant and
lactating) and children (3-6years) through Anganwadi centres (AWCs) along with the
Integrated Tribal Development Agencies (ITDA)
 The focus is on aspirational districts of Komaram-Bheem Asifabad, Bhadradri Kothagudem
and Jayashankar Bhupalpally in Telangana.
Conclusion Balanced/Futuristic based:
In conclusion, while India faces significant challenges in eradicating hunger, food insecurity, and
malnutrition, both the Centre and various states, including Telangana, are undertaking initiatives

TSPSC | Group-II | Batch-3 | Admissions in progress | Call 7013495019 or visit www.civiccentre.in


to address these issues. It is essential to continue to implement and strengthen these initiatives
to achieve the goal of eradicating hunger and malnutrition by 2030.
5. Discuss the role of NABARD with respect to the rural development.
Syllabus Paper III: Section III: Rural development
Keyword Discuss
Introduction Significance based:
NABARD came into existence on 12th July 1982 by transferring the agricultural credit functions of
RBI and refinance functions of the then Agricultural Refinance and Development Corporation
(ARDC). It was dedicated to the service of the nation by the late Prime Minister Smt. Indira Gandhi
on 05 November 1982.
Body NABARD (National Bank for Agriculture and Rural Development) is a development bank in India
that plays a crucial role in the overall rural development of the country. It was established with
the objective of promoting sustainable agriculture and rural development. Here are some of the
roles of NABARD in rural development:
Credit flow:
 NABARD plays a pivotal role in ensuring the smooth flow of credit to agriculture and rural
sectors.
 It provides refinance facilities to banks, cooperatives, and other financial institutions for
financing agriculture, rural development, and allied activities.
Capacity building:
 NABARD provides training and capacity building support to various stakeholders in rural
development, such as farmers, rural entrepreneurs, and NGOs.
 It also promotes the adoption of modern technology and best practices in agriculture and
allied activities.
Rural infrastructure development:
 NABARD provides financial assistance for developing rural infrastructure such as irrigation
systems, rural roads, markets, and warehouses.
 This infrastructure development helps to improve the overall agricultural productivity and
rural livelihoods.
Microfinance:
 NABARD plays a vital role in promoting microfinance in rural areas.
 It provides financial assistance to Microfinance Institutions (MFIs) and supports Self-Help
Groups (SHGs) and other community-based organizations to provide credit to rural
households and small entrepreneurs.
Promotion of sustainable agriculture:
 NABARD promotes sustainable agriculture practices such as organic farming, crop
diversification, and natural resource management.
 It also supports research and development in agriculture to enhance productivity and
sustainability.
Partnerships:
 It has built partnerships with other national entities, financial institutions and non-
governmental organizations in order to implement innovative ideas through loans,
guarantees, blended finance and other structures in the areas of agriculture, natural resources
management, fisheries, rural livelihood improvement, renewable energy and micro finance
among others.
Others:
 Almost one-third of its cumulative disbursements are related to climate change adaptation
and mitigation activities.

TSPSC | Group-II | Batch-3 | Admissions in progress | Call 7013495019 or visit www.civiccentre.in


 NABARD sought accreditation to the GCF in order to continue implementing its climate change
adaptation and mitigation projects and programmes, which are well aligned with the results
areas of the GCF, particularly food and water security, forestry and landscape management,
enhancing livelihoods and ecosystem services.
 Leveraging its long-standing partnerships and experience, NABARD intends to undertake low-
emissions and climate-resilient sustainable development that reduces the impacts of climate
change.
Current Context:
 With an aim to improve menstrual hygiene among women in rural areas, NABARD (National
Bank for Agriculture and Rural Development) launched the ‘My Pad My Right’ (MPMR) project
in Kokernag area of Anantnag.
 The SDPs are on-location training programmes with the objectives to provide a wide range of
skill development training to rural youth with job opportunities, identification and promote
existing business models for increased rural employment to benefit aspiring rural
entrepreneurs, improve income level and livelihoods.
 A NABARD-sponsored skill development programme (SDP) on sewing machine operator
for 30 unemployed rural youth and SHG members of Longding district was launched.
Conclusion Balanced/Based on Substantiation:
NABARD plays a crucial role in promoting sustainable rural development in India. Its interventions
have helped to improve the livelihoods of millions of people in rural areas by ensuring the smooth
flow of credit, building capacity, developing rural infrastructure, promoting microfinance, and
supporting sustainable agriculture practices.

TSPSC | Group-II | Batch-3 | Admissions in progress | Call 7013495019 or visit www.civiccentre.in


TSPSC Group-I |Naipunyata| A 100-Day Answer Writing Program
Day-74: Development Processes | Paper-III | Section- III| Unit-4
1. What is Public Private Partnership? Describe different models of PPP with examples
Syllabus Paper: III Section : III: Public Private Partnership (PPP)
Keyword What is and Describe
Introduction Definition based: PPP means an arrangement between a government or statutory entity or
government owned entity on one side and a private sector entity on the other,
 For the provision of public assets and/ or related services for public benefit
 Through investments being made by and/or management undertaken by the private
sector entity for a specified time period
 Where there is a substantial risk sharing with the private sector and the private sector
 Receives performance-linked payments that conform (or are benchmarked) to specified,
pre-determined and measurable performance standards.
 PPPs are required for infrastructure development and service delivery due to their ability
to leverage private sector financing, improve efficiency, manage risks, ensure
accountability, foster innovation, and promote public engagement.
Body Overview of PPP model variants
 Public-private partnerships (PPPs) can take a range of types encompassing various roles,
ownership arrangements, and allocations of risk between the private and public partners.
These different types are called PPP modes
 Different PPP modes can be compared on a spectrum ranging between low and high levels
of private participation and involvement. The four major “families'' of PPP modes are:
 Management contracts
 Lease contracts
 Concessions
 Build-operate-transfer (BOT) and its variants
Management Contract: This involves contracting to the private sector most or all of the
operations and maintenance of a public facility or service.
 Although the ultimate obligation of service provision remains with the public authority,
the day-to-day management control is vested with the private sector.
 Usually the private sector is not required to make capital investments.
These are prevalent in India across sectors. e.g., Hospital Beds in District Hospitals of
Telangana, Karnataka Urban Water Supply and Improvement Project, performance based
maintenance contracts in highways
 Private sector role: Management of all aspects of operation and maintenance:
Lease Contracts
Lease: Asset is leased, either by the public entity to the private partner or vice-versa.
 Private sector role: Management and maintenance
 Duration: Medium (e.g., 10-15yrs)
 e.g. Leasing of retail outlets at railway stations by Indian Railways
Build Lease Transfer (BLT) or Build-Own-Lease-Transfer (BOLT):
 BOT projects are typically large-scale, Greenfield infrastructure projects that would
otherwise be entirely funded, built and operated by the government.
 Involves building a facility, leasing it to the Government and transferring the facility after
recovery of investment.

TSPSC | Group-II | Batch-3 | Admissions in progress | Call 7013495019 or visit www.civiccentre.in


 Primarily taken up for railway projects such as gauge conversion in India in the past, with
limited success.
 Duration: Medium- (e.g. 10-15yrs)
 Capital investment focus & responsibility: Greenfield private
 Eg; A 6 lane national highway in India, a wastewater treatment facility in China, and a
power plant in the Philippines are just a few examples.
Build-Transfer-Lease (BTL):
 Involves building an asset, transferring it to the Government, and leasing it back. Here the
private sector delivers the service and collects user charges.
 Duration: Medium- (e.g., 10-15yrs)
 Capital investment focus & responsibility: Greenfield Private
 Examples: Chennai Trade centre, Mumbai-Pune Expressway etc
Concessions
 Responsibility for construction (typically brownfield / expansions) and operations with
the private partner while ownership is retained by the public sector.
 Herein the private sector (concessionaire) is responsible for the full delivery of services in
a specified area, including operation, maintenance, collection, management, and
construction and rehabilitation of the system. For example, water distribution
concessions for a city or area within the city.
 Duration: Long- (e.g. 20-30 yrs)
 Capital investment focus & responsibility: Brownfield/ Expansions
 Private partner role: Design, finance, construct, manage, maintain
Build-Operate-Transfer Contracts: Responsibility for construction (typically greenfield) and
operations with the private partner while ownership is retained by the public sector.
Design-build-operate (DBO):
 A private company or consortium is responsible for the design, construction, and
operation of the infrastructure asset for a defined period of time.
 The DBO model has been used extensively in India for various infrastructure projects,
including highways, airports, metro rail systems, and water supply projects.
 Private partner role: Design, construct, manage, maintain
 Duration: Short-medium- (e.g. 3-5 yrs)
 Examples: Hyderabad Outer Ring Road, Delhi-Meerut Expressway, Mumbai Metro Line 3
etc
Build-operate-transfer (BOT)/ Design-Build-Finance-Operate-Transfer (DBFOT):
 Under the DBFO model, the private company designs, builds, finances, and operates the
project for a specified period.
 Most common form of BOT concession in India.
e.g. Nhava Sheva International Container Terminal, Amritsar Interstate Bus
Terminal, Delhi Gurgaon Expressway, Hyderabad Metro, Salt Lake Water Supply and
Sewage Disposal System.
 Private sector role: Design, finance, construct, manage, maintain
 Duration: Long- (e.g. 20-30 yrs)
Build-operate-transfer (BOT) Annuity
 Under the BOT model, a private company builds and operates a project for a specified
period and transfers the ownership back to the public entity.

TSPSC | Group-II | Batch-3 | Admissions in progress | Call 7013495019 or visit www.civiccentre.in


 This has been adopted for NHAI highway projects in the past. More recently, it is the
preferred approach for socially relevant projects where revenue potential is limited.
e.g. Development of Kazipet Railway Line, Alandur Underground Sewerage Project
 Private partner role: Design, construct, manage, maintain
 Duration: Long- (e.g. 20-30 yrs)
Build-own-operate Transfer (BOOT) Contracts: Private partner has the responsibility for
construction and operations. Ownership is with the private partner for the duration of the
concession.
 Under the BOOT model, the private company builds, owns, and operates the project for
a specified period and then transfers the ownership back to the public entity.
 Examples of BOOT projects in India include Rajiv Gandhi International Airport (Hyd),
Mumbai International Airport, Bengaluru International Airport, and the Delhi Noida Toll
Bridge.
Build-own-operate (BOO)
 Build-Own-Operate (BOO) is a Public-Private Partnership (PPP) model that allows a
private company or consortium to design, finance, construct, own, and operate an
infrastructure asset for a specified period of time.
 The private partner assumes most of the risks and costs associated with the project and
typically earns revenue by selling the services or products produced by the infrastructure
asset.
 Example: Rooftop Solar Energy Panels(Renewable Energy Projects) in Telangana, Tirupur
Water Supply and Sewerage Project, Kempegowda International Airport etc
Hybrid Annuity Model (HAM)
 The HAM model is a combination of EPC (Engineering, Procurement, and Construction)
and BOT models.
 Under this model, the private company designs, builds, operates, and maintains the
project for a specified period, and the government pays the private company a fixed
amount every year.
 Examples of HAM projects in India include Kodad-Khammam National Highway
development by Adani Projects, Mumbai Nashik Expressway and the Gurgaon-Rewari
section of National Highway 8.
Conclusion Futuristic: Private participation is necessary for the success of Industry 4.0. Private companies
have the resources, expertise, and profit incentive to invest in and develop new technologies
that can drive the revolution forward. Private participation also fosters healthy competition,
flexibility, and job creation, all of which are crucial for the growth and success of Industry 4.0.
2. What is Corporate Social Responsibility? Do you think mandatory CSR hinder promotion of Industry?
Syllabus Paper V: Section I: Corporate Social Responsibility
Keywords What and Do you think
Introduction Definition based: Corporate Social Responsibility (CSR) refers to a company's commitment to
operate in an economically, socially, and environmentally sustainable manner. Under the
Companies Act, 2013, certain classes of profitable companies are required to spend at least 2
percent of their average net profit of the preceding three financial years on CSR activities in a
particular financial year.
 The Companies (Corporate Social Responsibility Policy) Amendment Rules, 2022 is one
such amendment that was introduced on September 20, 2022

TSPSC | Group-II | Batch-3 | Admissions in progress | Call 7013495019 or visit www.civiccentre.in


 Companies are required to establish a CSR committee to monitor the execution of their
CSR commitments and in particular any funds in their “Unspent Corporate Social
Responsibility Account”
Body Statutory backing of CSR
 The Corporate Social Responsibility concept in India is governed by Section 135 of the
Companies Act, 2013 (‘Act’), Schedule VII of the Act and Companies (CSR Policy) Rules,
2014.
 Be it a private sector company or a public sector company, Corporate Social
Responsibility CSR has to be adhered to by all listed companies to spend 2% of their
average net profits of three years on CSR
 If a company falls in either of the following criteria for compulsion, they need to form
a CSR committee. Companies:
 That has a net worth of Rs. 500 crores or more, or
 That have an annual turnover of Rs. 1000 crores or more, or
 That generate a profit of Rs. 5 crores or more.
Activities covered in Schedule VII of Companies Act 2013
 Eradicating hunger, poverty and malnutrition; promoting health care including preventive
health care and sanitation
 Promoting education, including special education and employment enhancing vocational
skills especially among children, women, elderly, and the differently abled and livelihood
enhancement projects
 Promoting gender equality and empowering women: setting up homes and hostels for
women and orphans; setting up old age homes, day care centers and such other facilities
for senior citizens and measures for reducing inequalities faced by socially and
economically backward groups
 Training to promote rural sports, nationally recognized sports, Paralympic sports and
Olympic sports
 Protection of national heritage, art and culture including restoration of building and sites
of historical importance and works of art etc
 Rural development projects and slum area development
Companies in India contributing to CSR: TCS, Wipro, AMUL, Café Coffee Day, Reliance etc
How a company can undertake CSR activity
 CSR activities must be undertaken by a separate entity - it could be a trust, a society or
another company set up specifically for the purpose.
 It is not necessary that these trusts, societies and companies should be set up by the
company on its own
 Companies can work with an existing independent one that has a three-year track record
for carrying out such activities.
 Companies can also collaborate with other companies for a joint CSR programme -
provided each can identify and report separately their part of the joint activity and what
was achieved.
How CSR can reduce the amount of taxation
 Money spent on all corporate social responsibility (CSR) works cannot be counted as
business expenditure but certain social welfare spending activities could be considered
for tax benefits, according to Central Board of Direct Taxes (CBDT)
 The expenses incurred on CSR activities are tax-deductible, up to a limit of 10% of the
company's total income.

TSPSC | Group-II | Batch-3 | Admissions in progress | Call 7013495019 or visit www.civiccentre.in


 Exemption from Minimum Alternate Tax (MAT): Companies that claim tax deductions for
CSR expenses may be exempted from paying Minimum Alternate Tax (MAT)
Promotion of development indicators in turn helping the industry
 Education and Skill Development: Companies that invest in education and skill
development initiatives, such as vocational training programs or scholarship schemes, can
contribute to building a skilled workforce that is essential for the growth of the industry.
 A skilled workforce can increase productivity, reduce costs, and improve the quality
of products and services.
 Health and Sanitation: Companies that undertake health and sanitation initiatives, such
as building public toilets or providing clean drinking water, can contribute to improving
the health and well-being of the local community.
 A healthy workforce can increase productivity and reduce absenteeism, which can
benefit the industry.
 Environmental Sustainability: Companies that invest in environmental sustainability
initiatives, such as renewable energy or waste management, can contribute to creating a
sustainable business environment that supports long-term growth.
 Environmental sustainability can reduce costs, improve brand image, and attract
customers who are environmentally conscious.
 Infrastructure Development: Companies that undertake infrastructure development
initiatives, such as building roads or bridges, can contribute to improving the connectivity
and accessibility of the local community.
 Improved infrastructure can attract investment, create job opportunities, and
support economic growth, which can benefit the industry.
 Gender Equality and Diversity: Companies that promote gender equality and diversity in
the workplace, such as providing equal opportunities for women and men, can create a
more inclusive and supportive work environment.
 This can help attract and retain talented women in the industry, which can benefit
the industry by promoting innovation and creativity.
 Entrepreneurship and Microfinance: Companies that support entrepreneurship and
microfinance initiatives for women, such as providing access to credit and business
training, can help them start and grow their own businesses in the industry.
 This can promote economic empowerment, increase their participation in the
industry, and contribute to economic growth.
Conclusion Futuristic based: In conclusion, investing in CSR can have significant returns on investment by
enhancing brand reputation, increasing employee engagement and productivity, reducing
operational costs, mitigating business risks, and accessing new markets and customers.
3. What are the challenges faced by SHGs in India? How can those challenges be tackled to promote
participatory development in India?
Syllabus Paper V: Section I: Self Help Groups
Keywords What and How
Introduction Importance/Significance based: Self Help Groups (SHGs) in India are typically groups of 10-20
people who come together to form a collective, mutually supportive social and financial
network. These groups are often formed with the objective of improving the socio-economic
status of their members, particularly women, through savings, credit, and other social and
economic activities. While there is no specific legislation that governs the functioning of SHGs
in India, there are various government policies and schemes that provide statutory backing
for their regularisation and support.

TSPSC | Group-II | Batch-3 | Admissions in progress | Call 7013495019 or visit www.civiccentre.in


Body Challenges faced by SHGs
 Financial: Most SHGs operate with limited financial resources and struggle to access
formal credit facilities.
 This limits their ability to undertake large-scale economic activities and to expand
their operations.
 Capacity building: Many SHGs lack the necessary skills and knowledge to manage their
financial affairs, market their products effectively, and access government schemes and
programs.
 This makes it difficult for them to sustain their operations and achieve their goals.
 Social: SHGs, particularly those formed by women, often face social barriers and
prejudices that hinder their growth and development.
 These include discrimination, gender bias, and patriarchal attitudes.
 Governance: Many SHGs lack strong governance structures and mechanisms to ensure
accountability and transparency in their operations.
 This can lead to mismanagement of funds and other resources, and can undermine
the trust and confidence of members.
 Infrastructure: SHGs often lack access to basic infrastructure such as roads, electricity,
and water, which makes it difficult for them to engage in economic activities and to
market their products.
 Legal: Many SHGs face legal challenges related to their registration, compliance with
government regulations, and protection of their rights and interests.
 This can create uncertainty and instability in their operations, and can limit their
ability to access formal credit and other resources.
 Technology: SHGs often lack access to technology and information, which limits their
ability to compete in the market and to access new markets and customers.
Measures to overcome those challenges
Individual level measures:
 Capacity-building: Members of SHGs need to be trained and educated in various skills
such as financial management, marketing, and entrepreneurship. This will enable them
to manage their businesses effectively and efficiently.
 Awareness raising: Members of SHGs need to be made aware of their legal and social
rights, and how to access various government schemes and programs.
 Leadership training: Members of SHGs need to be trained in leadership and decision-
making, which will enable them to manage their group effectively and represent their
interests in various forums.
 Personal development: Members of SHGs need to be provided with opportunities for
personal development, such as training in communication and public speaking.
Societal level measures:
 Addressing social barriers: Society needs to address social barriers such as gender
discrimination, caste biases, and patriarchal attitudes that hinder the growth and
development of SHGs, especially those formed by women.
 Encouraging entrepreneurship: Society needs to create a culture that encourages
entrepreneurship and supports the growth of small businesses, which will benefit SHGs.
 Access to infrastructure: Society needs to ensure that SHGs have access to basic
infrastructure such as roads, electricity, and water, which will enable them to engage in
economic activities and to market their products.

TSPSC | Group-II | Batch-3 | Admissions in progress | Call 7013495019 or visit www.civiccentre.in


Organizational level measures:
 Capacity-building: SHGs need to be provided with capacity-building support to manage
their financial affairs, market their products effectively, and access government schemes
and programs.
 Strengthening governance: SHGs need to develop strong governance structures and
mechanisms to ensure accountability and transparency in their operations, and to protect
the interests of their members.
 Access to credit: SHGs need to be provided with access to formal credit facilities, which
will enable them to undertake large-scale economic activities and to expand their
operations.
 Use of technology: SHGs need to be provided with access to technology and information,
which will enable them to compete in the market and to access new markets and
customers.
 In order to solve the various problems relating to marketing of SHGs, the state level
organisations should extend the activities throughout the state instead of limiting its
operations in a particular area.
 All the members in the SHGs may not have the same calibre and expertise. NGOs can
identify the inefficient members of the group and can impart proper training to them in
order to make them competent through short-term training programmes at the
panchayat level.
Conclusion SDG- Based:
SHGs can contribute to the achievement of SDG5 by achieve gender quality and empower all
women and girls and increase their decision-making in financial issues.
4. Discuss the role of non-governmental organisations in promoting social welfare among vulnerable sections
of society.
Syllabus Paper V: Section I: Non-governmental Organisations
Keyword Discuss
Introduction Definition based: A Non-Governmental Organization (NGO) is a non-profit, voluntary
organization that operates independently of the government and is primarily focused on
promoting social, cultural, environmental, or political objectives.
Eg., Examples of NGOs include the Red Cross, Amnesty International, Greenpeace, and the
Self-Employed Women's Association (SEWA).
NGOs like CRY (Child Rights and You) and Save the Children work towards ensuring that
children from marginalized communities have access to basic education, healthcare, and
nutrition
Body Constitutional provision for NGOs:
 Article 43 emphasises the state's objectives and the aim of fostering rural cooperatives.
The freedom to create associations is covered under Article 19 (1)(c).
 In Entry 28, the Concurrent List includes Charitable Institutions, Charity, Religious and
Charitable endowments, and Religious Songs Institutions.
Role of NGOs in bringing problems of vulnerable sections of the society
Dalit Rights:
 NGOs such as the National Campaign on Dalit Human Rights, the Centre for Tribal and
Rural Development, and the Adivasi Adhikar Manch have been working to address the
problems faced by Dalits and tribals in India.
 They have been advocating for land rights, access to education and healthcare, and
protection from discrimination and violence.

TSPSC | Group-II | Batch-3 | Admissions in progress | Call 7013495019 or visit www.civiccentre.in


Women's Rights:
 NGOs such as the All India Women's Conference, Breakthrough India, and Lawyers
Collective have been working to promote women's rights and gender equality in India.
 They have been advocating for stronger laws and policies to protect women from
violence, harassment, and discrimination. They also provide support to survivors of
violence and work to build the capacity of women to assert their rights.
LGBTQ+ Community:
 NGOs such as the Humsafar Trust, Naz Foundation, and the Lawyers Collective have been
working to promote the rights of the LGBTQ+ community in India.
 They have been advocating for the decriminalization of homosexuality, the right to
marriage and adoption, and access to healthcare and other services.
Children:
 NGOs such as Save the Children, Child Rights and You (CRY), and Bachpan Bachao Andolan
have been working to protect the rights of children in India.
 They have been advocating for access to education, protection from child labor and
trafficking, and the right to a safe and healthy childhood.
How NGOs work for the vulnerable sections of society
Healthcare:
 NGOs like the Indian Cancer Society, HelpAge India, and the Smile Foundation work
towards providing healthcare services to vulnerable sections of society.
 These organizations run free medical camps, provide access to medication, and create
awareness about preventive healthcare measures.
Education:
 NGOs like Pratham and Teach for India work towards improving access to education for
children from marginalized communities.
 They provide remedial classes, support teachers, and promote technology-enabled
learning to improve the quality of education in schools.
Women Empowerment:
 NGOs like Self Employed Women's Association (SEWA) and the Guild for Service work
towards empowering women by providing them with education and training, access to
finance, and creating job opportunities.
 They also work towards raising awareness about women's rights and gender-based
violence.
Conclusion Balanced/Futuristic based: NGOs are often seen as a watchdog, ensuring that the government
and other institutions act in accordance with the law and the best interests of society.
Amnesty's exit from India is just one example of the challenges faced by NGOs in the country.
Many NGOs have been subject to increased scrutiny and restrictions in recent years, with the
government accusing them of violating laws and engaging in anti-national activities. However,
the challenges faced by NGOs in India highlight the need for greater protection of civil society
and the need for greater respect for the rights and freedoms of NGOs.
5. Civil society plays a significant role in India in promoting democratic values, social justice, and human
rights. Examine
Syllabus Paper III: Section III: Civil Society
Keyword Discuss
Introduction  Civil society means the entire range of organized groups and institutions that are
independent of the state, voluntary, and at least to some extent self-generating and self-

TSPSC | Group-II | Batch-3 | Admissions in progress | Call 7013495019 or visit www.civiccentre.in


reliant. This includes non-governmental organizations like the ones in this room, but also
independent mass media, think tanks, universities, and social and religious groups.
 For democratic state to function well, presence of a strong civil society is a must.
Body Role of civil society:
Advocacy and Awareness Campaigns
 Civil society organizations in India raise awareness about democratic values and the
importance of citizen participation in the democratic process.
 They advocate for policies that promote transparency, accountability, and good
governance and provide training and support for citizens to participate in the democratic
process.
Election Monitoring:
 Civil society groups in India often play a significant role in monitoring elections and
ensuring that they are free and fair.
 They observe the election process, report any irregularities, and work to hold the
government accountable for any violations of democratic norms.
Advocacy for Political Reforms:
 Civil society groups in India advocate for political reforms that promote democratic values,
such as electoral reforms, campaign finance reform, and the decentralization of power.
 They work to ensure that the political system is more inclusive, transparent, and
accountable.
Citizen Participation
 Civil society organizations in India work to promote citizen participation in the democratic
process.
 They encourage citizens to vote, engage with their elected representatives, and hold them
accountable for their actions.
 They also work to ensure that marginalized communities have equal access to the
democratic process.
Examples:
 Civil society played a key role in Jan Lokpal Andolan to force government to frame statute
to establish office of Lokpal
 Kisan shakti sangathan played a crucial role to culminate in granting right to information
act
Role of civil society in promoting social justice
 As a society, we need civil societies like anything because it will protect the basic human
needs against the tyranny of the state.
 The rights of the minority, depressed class along with individual rights of freedom, liberty
are protected duly by civil society.
 Further, civil society has a great role in stopping the majoritarian tendencies and
protecting the spirit of diversity in India.
 Although corruption has not died down completely, the awareness due to the civil society
movement has affected the corrupt structure greatly.
Examples
 NGOs like ‘Asha,’ ‘Pratham’ are actively involved in providing education to the children in
rural areas and slums in the country.
 NGO like ‘Rural Health Care Foundation’ are providing health care services to the people
in rural and remote areas.

TSPSC | Group-II | Batch-3 | Admissions in progress | Call 7013495019 or visit www.civiccentre.in


 Be it the movement for freedom of expression, be it for the fight of LGBT community
against the decriminalisation under section 377 of IPC, or be it the fight of women against
instant triple talaq there is present the active contribution and wholehearted support of
civil society.
Role of Civil society in safeguarding Human rights:
 Civil society organizations perform important research to help Governments understand
and respond to problems and needs on the ground.
 They monitor the work of courts to ensure that victims’ voices are heard and the rights of
all are respected.
 They can empower community members with knowledge of the law, including laws that
are essential to protecting the environment, preventing corruption, and upholding human
rights.
 Civil society organizations in India have also been involved in providing direct assistance
to victims of human rights abuses.
 They have provided legal aid, counselling, medical and rehabilitation services, and other
forms of support to victims of human rights violations.
Examples:
 The National Campaign for Dalit Human Rights (NCDHR) has been advocating for the rights
of Dalits, who face discrimination and violence because of their caste.
 The All India Women's Conference (AIWC) has been working to promote the rights of
women in India.
Issues facing civil society:
 Civil society organizations lack adequate financial resources.
 Most of the CSOs don’t have adequate funds to carry out their work, as the people
they serve lack the capacity to pay.
 Hence, they depend on receiving funds from the government/non-government
bodies/international institutions. Now, with a simultaneous cut in the international
funding, FCRA regulations situations organisations are starving.
 Accountability
 There have been increasing incidences of misuse of funds by these organizations. In
January 2017
 the Hon. The Supreme Court of India called for an audit of nearly 30 lakh NGOs. This
move was taken in the background of the NGOs failing to give an account of
expenditure made out of the money they had received.
 Inadequate professional and trained staff
 There exists a huge shortage of professional and trained personnel in the voluntary
sector. Most of the personnel are unqualified and unskilled. Inadequate salaries do
act as a deterrent in appointing a skilled workforce.
 Also, professionally qualified and trained individuals have a preference and
inclination to work in high profile cities and offices rather than a backward and
underdeveloped region.
Conclusion Futuristic based: Civil society is necessary for achieving growth and development as well as
the SDGs. Their advocacy, monitoring, and accountability functions, innovative solutions, and
community engagement can contribute to more effective and sustainable development
outcomes. Civil society organizations can play a vital role in advancing progress towards the
SDGs and promoting inclusive growth and development that benefits all members of society.

TSPSC | Group-II | Batch-3 | Admissions in progress | Call 7013495019 or visit www.civiccentre.in


TSPSC Group-I |Naipunyata| A 100-Day Answer Writing Program
Day-75: Ethics and Values of Administration | Paper-III | Section-III | Unit-V
1. Bring out various Ethical values in Public Service mentioned by different committees
Syllabus Paper III: Section III: Ethics and Values of Administration
Keyword Bring Out
Introduction  Brief Introduction about the importance of ethical values that need to follow during public
service stating that people in public service generally work to improve the lives of the
public, their behaviour in public service is based on subjective morals and belief systems
as opposed to universal, ethical principles.
Body  Various committees have recommended ethical values for public servants to inculcate.
Two of these important committees include the Nolan Committee and the 2 nd
Administrative Reform Commission (ARC).
 Nolan Committee on Standards in Public Life in the United Kingdom, popularly known as
the Nolan Committee, outlined the following
seven principles of public life:
 Selflessness: Holders of public office
should take decisions solely in terms of
public interest. They should not do so in
order to gain financial or other material
benefits for themselves, their family or
their friends.
 Integrity: Holders of public office should
not place themselves under any financial
or other obligation to outside individuals
or organizations that might influence
them in the performance of their official
duties.
 Objectivity: In carrying out public business, including making public appointments,
awarding contracts or recommending individuals for rewards and benefits, holders of
public office should make choices on merit.
 Accountability: Holders of public office are accountable for their decisions and
actions to the public and must submit themselves to whatever scrutiny is appropriate
to their office.
 Openness: Holders of public office should be as open as possible about all the
decisions and actions they take. They should give reasons for their decisions and
restrict information only when the wider public interest clearly demands.
 Honesty: Holders of public office have a duty to declare any private interests relating
to their public duties and to take steps to resolve any conflicts arising in a way that
protects the public interest.
 Leadership: Holders of public office should promote and support these principles by
leadership and example
 As per 2nd ARC recommendations, there should be a clear and concise statement of the
values and ethical standards that a civil servant should imbibe. These values should reflect

TSPSC | Group-II | Batch-3 | Admissions in progress | Call 7013495019 or visit www.civiccentre.in


public expectations from a civil servant with reference to political impartiality,
maintenance of highest ethical standards and accountability for actions.
 In addition to commitment to the Constitutional values, the 2nd ARC recommended the
following ethical values which include:
 Adherence to the highest standards of probity, integrity and conduct
 Impartiality and non-partisanship
 Objectivity
 Commitment to the citizens' concerns and public good
 Empathy for the vulnerable and weaker sections of society.
Conclusion Substantiation based:
Thus, Ethical values are an essential part of public service work to maintain the integrity of
democratic institutions.
2. Compare and contrast between Code of Ethics and Code of Conduct.
Syllabus Paper III: Section III: Ethics and Values in Administration
Keyword Compare and Contrast
Introduction Definition based:
 Brief introduction by stating about what is Code of ethics and Code of Conduct
 Code of ethics includes core ethical values, principles and ideals of the organisation
 Code of Conduct is a set of rules, standards, principles and values outlining the
expected behaviour for the members of an organisation.
Body  Code of Ethics
 It is referred to as values, which behaves like the Constitution with general principles
to guide behaviour, outlining a set of principles that affect decision-making
 It includes the principles of integrity, impartiality, commitment to public service,
accountability, devotion to duty, exemplary behaviour etc
 Code of Conduct
 It is a document containing specific practices and behaviour, that are followed or
restricted under the organisation
 These are legally enforceable for setting out standards of behaviour in public life.
Similarities between Code of Ethics and Code of Conduct
 Both are meant for to guide behaviour of a public servant.
 Both are focused on making governance more ethical.
 Basis of Code of Conduct is emanated from Code of Ethics.
 Both avoid or handle situations like conflict of interests, dilemmas, debatable
situations etc.
Comparison between Code of Ethics and Code of Conduct
Code of Ethics Code of Conduct
 Aspirational: to set aspirational  Directional – to provide guidance on
standards that all are expected to acceptable and unacceptable
live up to conduct and practices
 General and Broad in nature  Specific and narrow in nature
 Primary focus: values and principles  Primary focus: guidelines or rules for
as a basis for behaviour behaviour

TSPSC | Group-II | Batch-3 | Admissions in progress | Call 7013495019 or visit www.civiccentre.in


 A proactive approach to provide  A reactive approach to ensure
values-based reasons for good compliance with prescribed
behaviour standards of conduct
 Promotes ethical behaviour: setting  Prevents unethical behaviour:
ethics standards that contribute to provides behavioural guidelines; a
ethical culture formation necessary but not sufficient
condition for ethical culture
formation.
 Help and guide decision making at  Prevents certain types of behaviours
the time of ethical dilemmas or like conflict of interest, self-dealing,
debatable situations bribery and inappropriate actions
 Disciplinary power: Should never  Disciplinary power: Has punitive
have a punitive intent; does not powers; may be tabled during a
belong in a disciplinary disciplinary inquiry/hearing.
inquiry/hearing

Conclusion Based on Substantiation:


Conclude by stating that while code of conduct is something that states organized values,
code of ethics is used to observe ethical norms and gives a foundation to rules of conduct.
3. The Right to Information Act of 2005 brought a radical shift in our governance culture. Discuss
Syllabus Paper III: Section III: Right to Information Act 2005
Keywords Discuss
Introduction Definition/Significance based:
Brief introduction about the background of Right to Information Act of 2005.
 Right to information have roots in Universal Declaration of Human Rights of 1948
providing everyone the right to seek, receive, information and ideas through any media
and regardless of frontiers.
Body  The right to information is a fundamental right under Article 19 (1) of the Indian
Constitution.
 As per Supreme Court, people are the masters and they have the right to know about the
working of the government.
 Right to Information act was enacted in 2005 which provided mechanism for exercising
this fundamental right.
 The act mandates public authorities of union government or state government, to provide
timely response to the citizens’ request for information.
 All constitutional authorities, agencies, owned and controlled, also those
organisations which are substantially financed by the government comes under the
purview of the act.
 The act also imposes penalties if the authorities delay in responding to the citizen in the
stipulated time.
 The citizens can seek any information from the government authorities that the
government can disclose to the parliament.
 Some information that can affect the sovereignty and the integrity of India is
exempted from the purview of RTI.

TSPSC | Group-II | Batch-3 | Admissions in progress | Call 7013495019 or visit www.civiccentre.in


 Information relating to internal security, relations with foreign countries, intellectual
property rights (IPR), and cabinet discussions are exempted from RTI.
 Objectives of the RTI Act:
 It empowers citizens to question the government.
 The act promotes transparency and accountability in the working of the government.
 The act also helps in containing corruption in the government and work for the people
in a better way.
 The act envisages building better-informed citizens who would keep necessary vigil
about the functioning of the government machinery.
 The changes brought by the act in governance are
 It empowered the citizen to question the secrecy and abuse of power practised in
governance.
 The information obtained through the act made the government accountable.
 It enabled the people’s participation in administration where around six million
applications are filed under the RTI Act every year.
 It empowered citizens in a way that even the highest offices of the country were
questioned.
 People have sought information that governments would not like to reveal as it may
expose corruption, human rights violations, and wrongdoings by the state.
 Accountability was ensured through transparency in the functioning the
Governments.
 Way Forward
 Protection mechanism for the interest of whistle-blowers
 Protect the autonomy and independence of State and Central information
commissioners.
 Increase the staff strength for smooth functioning of information commissions.
Conclusion Balanced/Based on Substantiation/Futuristic based:
Conclude by stating that RTI provides us with a priceless opportunity to redesign the processes
of governance, particularly at the grassroots level where the citizens’ interface is maximum.
4. Give a brief account of Prohibition of Benami Property Transaction Act, 1988.
Syllabus Paper III: Section III: Prevention of corruption in administration
Keyword Give a brief Account of
Introduction Definition/Report/Context/Significance based:
Brief introduction about the term Benami transaction
 Benami transaction means any transaction in which property is transferred to one person
for a consideration paid or provided by another person.
Body  A transaction is considered benami (literally ‘nameless’ or ‘without a name’) when the
consideration for a property that is transferred to a person or is held by him/her is paid
by another person.
 In such transactions, the person who pays for the property is the ultimate beneficiary
of the property, directly or indirectly, in the future.
 A Benami Transactions (Prohibition) Act was passed in 1988 to put an end to such
transactions, and to empower the government to recover such property.

TSPSC | Group-II | Batch-3 | Admissions in progress | Call 7013495019 or visit www.civiccentre.in


 The Act, however, had several inherent loopholes — including the absence of an appellate
mechanism, and lack of provisions for vesting of the confiscated property with the Centre.
Owing to these infirmities, the Rules for the Act were also not framed.
 Recently the Benami Transaction (Prohibition) Act 1988 has been amended and renamed
it as Benami Transactions Prohibition (Amendment) Act, 2016.
 It prescribes the punishment for entering into benami transaction.
 Confiscation of benami property
 It extended the three-year imprisonment to seven years and fine of up to 25% of the
fair market value of the property, if a person enters into any benami transactions.
 The Act defines a benami transaction as a transaction where:
 A property is held by or transferred to a person, but has been provided for or paid by
another person.
 The transaction is made in a fictitious name
 The owner is not aware of denies knowledge of the ownership of the property,
 The person providing the consideration for the property is not traceable.
 The Act provides for an Appellate Tribunal to hear appeals against any orders passed by
the Adjudicating Authority.
 The Act established four authorities to conduct inquiries or investigations regarding
benami transactions:
 Initiating Officer- To issue a notice to benamidar
 Approving Authority- has authority to pass an order on whether or not to hold the
property as benami based on assessment.
 Administrator-
 Adjudicating Authority
 Supreme Court had struck down some provisions of the Benami law.
 One of the struck down provisions provided for the punishment of a maximum jail
term of three years or a fine or both for those indulging in ‘benami’ transactions.
 Supreme Court also held that the amended Benami law of 2016 did not have retrospective
application and the authorities cannot initiate or continue criminal prosecution or
confiscation proceedings for transactions entered into prior to the coming into force of
the legislation.
Conclusion Balanced/Based on Substantiation/Futuristic based:

Thus, Prohibition of Benami Property Transaction Act, 1988 is a right step towards the
elimination of black money and thus reducing corruption in Indian economy.

5. What is the importance of accountability and ethical governance in public administration


Syllabus Paper III: Section III: Ethics and Accountability of Administration
Keyword What
Introduction Definition/Report/Context/Significance based:
Brief introduction by stating that accountability and ethical governance are essential for the
effective functioning of public administration.
Body  Trust:
 Accountability and ethical governance are essential for building trust between the
government and its citizens.

TSPSC | Group-II | Batch-3 | Admissions in progress | Call 7013495019 or visit www.civiccentre.in


 When the government is transparent and accountable for its actions, citizens are
more likely to trust its decisions and actions.
For example, through Direct Benefit Transfer if the amount is directly transferred to the
beneficiary account, then it builds trust on the administrative machinery
 Fairness:
 Accountability and ethical governance ensure that public administration is conducted
in a fair and equitable manner.
 It ensures that decisions are made based on objective criteria, rather than personal
biases or interests.
For example, if a Public Service Commission have conducted examination and had put out the
examination OMRs and marks online which is visible to everyone, it ensures fairness of the
recruitment process
 Efficiency:
 When public administration is conducted with accountability and ethical governance,
it tends to be more efficient.
 This is because resources are used more effectively and decisions are made more
quickly, based on clear and transparent criteria.
For example, if administrative machinery calls for Tenders to implement the projects through
any of the PPP modes, then it ensures government’s revenue is utilised effectively
 Public participation:
 Accountability and ethical governance encourage public participation in decision-
making processes.
 When citizens feel that their voices are being heard and that the government is being
held accountable, they are more likely to participate in governance processes.
For example, Gram Sabha playing a huge role in the social audit of government schemes
 Prevention of corruption:
 Accountability and ethical governance are crucial for preventing corruption in public
administration.
 When the government is transparent and accountable, there is less opportunity for
corruption to take hold.
For example, if the government is appointing LokPal and Lokayukta promptly and ensure
effective resources to discharge its duties, then the government’s intent to prevent corruption
can be clearly visible
 Compliance with laws and regulations:
 Accountability and ethical governance ensure that public administration is conducted
in compliance with laws and regulations.
 This helps to prevent abuses of power and ensures that the government operates
within the framework of the law.
For example, effective utilisation of PIL to scrutinise the laws framed by the Legislature under
Article-13 or 226 can ensure that due process of law is followed
Conclusion Balanced/Based on Substantiation/Futuristic based:
Conclude by stating that accountability and ethical governance are important for building
trust, promoting fairness, increasing efficiency, encouraging public participation, preventing
corruption, and ensuring compliance with laws and regulations in public administration.

TSPSC | Group-II | Batch-3 | Admissions in progress | Call 7013495019 or visit www.civiccentre.in


TSPSC Group-I | Mains Test Series
Test-21: Governance | Paper-III | Section-III | Model Answers
1. Discuss the features of Good Governance with suitable examples.

Approach:

1. Identify the keyword “Discuss”


2. Introduction: Context/Example
3. Body: Mentions various features with suitable examples
4. Conclusion

Model Answer:

Black money to the tune of “₹ 1.25 lakh crore” has been confiscated by the Modi government, Union Minister
Ashwini Vaishnaw said recently and asserted transparency is a key aspect of the present dispensation’s good
governance model. According to World Bank (1997), “Good Governance is, among other things, participatory,
transparent and accountable. It is also effective and equitable. And it promotes the rule of law. Good
governance ensures that political, social and economic priorities are based on broad consensus in society and
that the voices of the poorest and the most vulnerable are heard in decision-making over the allocation of
development resources.”

Features of Good Governance:

There are eight important features of good governance. It is viewed that these features assure minimisation
of corruption and the views of minorities and most vulnerable in society are taken care of in decision-making.

1. Accountability:

Accountability means being able to provide an explanation or justification and accept responsibility, for
events or transactions and for one's own actions in relation to these events or transactions. Accountability
plays a particularly important role in the public sector. Decision makers in government, the private sector
and civil society organizations are accountable to the public, as well as to institutional stakeholders.

Elections are the main mechanism through which citizens are given the power to ensure accountable
governance. Citizens make their voting decisions and either vote the government back into power or out
of it depending on their evaluation of whether it delivered on its promises during its term. Accountability
claimed through this channel has been called vertical accountability. There are also provisions for
horizontal accountability institutional checks and balances to ensure that the government is performing
its duties in the interest of its citizens.

2. Participation:

All the citizens should have a voice in decision-making, either directly or through intermediate institutions
that represent their interests. Such participation is built on freedom of association and speech, as well as
capacities to participate constructively.

Citizens are important stakeholders and play a critical role in advocating and enabling public institutions
to become more transparent, accountable, and effective and suggest innovative solutions to complex
development challenges. Citizen engagement is at the core of good governance. Mygov.in is a platform to
engage citizens and get their input and suggestions for various government policies and plans in India.
MyGov platform provides an opportunity to citizens across the world to engage directly with the
government departments, policymakers, and implementers. MyGov is planned as the key platform for all

TSPSC | Group-2 | Batch-3 | Admissions in progress | Call 7013495019 for admissions


citizen engagement needs of the country across various departments and ministries. The presence of such
a digital platform in a democratic country reflects willingness on part of the government to share
information and make citizens a partner in decision making

3. Transparency:

It is built on the free flow of information. Processes, institutions and information are directly accessible to
those concerned with them, and enough information is provided to understand and monitor them. It is
also important for ensuring accountability and enabling genuine participation.

Transparency is required to ensure the effectiveness of the public service delivery system. However,
simply knowing what entitlements are and who is responsible for fulfilling them is insufficient to ensure
that public services are delivered in a passable and effective manner to the ‘intended’ recipients.

Citizens Charters, the Right to Information Act, e-procurement, e-Governance, and Right to Public Service
Legislation, are some of India’s major initiatives to improve transparency.

4. Responsiveness:

Institutions and processes try to serve all stakeholders. Responsiveness describes the extent to which a
public service agency demonstrates receptivity to the views, complaints and suggestions of service users,
by bringing about and implementing changes to its own structure, culture, and service delivery patterns
in order in deliver a more appropriate product.

Most state bureaucracies are in fact ‘responsive’ but to socially powerful interest groups, not the poor.
Promoting responsiveness to a broader range of social groups and particularly to the poor and other
socially excluded groups can involve promoting counter-cultural reforms in bureaucratic behaviour

5. Rule of law:

Legal frameworks should be fair and enforced impartially, particularly the laws on human rights. Indian
Constitution is considered to be supreme and no one is above Indian Constitution. Rule of law is also given
impliedly in the preamble and such concept is enshrined in Part III of the Indian Constitution.

In case of violation of such rights, one can approach Supreme Court or High Court under Article 32 and
226 of the Indian Constitution. The Constitution of India is enriched with the principles of law i.e. justice,
equality and liberty.

6. Consensus:

Good governance mediates differing interests to reach a broad consensus on what is in the best interests
of the groups and on policies and procedures. It also requires a broad and long-term perspective on what
is needed for sustainable human development and how to achieve the goals of such development. This
can only result from an understanding of the historical, cultural and social contexts of a given society or
community.

7. Equity and Inclusiveness:

All men and women and different social groups have opportunities to improve or maintain their well-
being. A society’s well-being depends on ensuring that all its members feel that they have a stake in it and
do not feel excluded from the mainstream of society. This requires all groups, but particularly the most
vulnerable, have opportunities to improve or maintain their well-being. The Mahatma Gandhi National
Rural Employment Guarantee Act, 2005, provides employment opportunities to rural households,

TSPSC | Group-2 | Batch-3 | Admissions in progress | Call 7013495019 for admissions


promoting inclusiveness and reducing poverty. Also, the reservation system helps in providing equity to
the economically and socially backward classes.

8. Effectiveness and efficiency:

Processes and institutions produce results that meet the needs while making the best use of resources.
Hence, the concept of efficiency in the context of good governance also covers the sustainable use of
natural resources and the protection of the environment. The Digital India initiative aims to provide all
citizens with digital access to government services, promoting efficiency and effectiveness in service
delivery.

Good governance is essential to ensure the effective and efficient delivery of public services, protecting
citizens' rights and interests, and promoting economic development. The features of good governance,
including the rule of law, transparency and accountability, participation and inclusiveness, efficiency and
effectiveness, and responsiveness, are critical to achieving these goals.

2. "e - Governance is not merely use of information technology in enabling governance, but an effort to achieve
good governance." Comment.

Approach:

1. Identify the keyword “Comment”


2. Introduction: Definition/Significance based.
3. Body: How e-governance ensures good governance
4. Conclusion:

Model Answer:

E-governance, or electronic governance, is the use of information technology to provide better and more
efficient public services to citizens. According to the World Bank, e-governance refers to the use by
government agencies of information technologies such as Wide Area Networks, the Internet, and mobile
computing that have the ability to transform relations with citizens, businesses, and other arms of
government. However, it is not just about the use of technology but also about the transformation of the
entire governance process to ensure good governance.

Over the years, a large number of initiatives have been undertaken by various State Governments and Central
Ministries to usher in an era of e-Government. Sustained efforts have been made at multiple levels to improve
the delivery of public services and simplify the process of accessing them.

Good governance refers to the effective and efficient delivery of public services, transparency, and
accountability, rule of law, participation, and responsiveness to citizens' needs. E-governance can enable good
governance by providing citizens with access to information, improving service delivery, increasing
transparency, and reducing corruption.

For instance, E-Courts aim to provide efficient & time-bound citizen centric services delivery; to develop, install
& implement decision support systems in courts; to automate the processes to provide transparency in
accessibility of information to its stakeholders; to enhance judicial productivity, both qualitatively &
quantitatively, to make the justice delivery system affordable, accessible, cost effective, predictable, reliable
and transparent.

e-Districts are the de facto front-end of government where most Government-to-Consumer or G2C interaction
takes place. To improve this experience and enhance the efficiencies of the various Departments at the
district-level, e-District project was envisaged to enable providing of integrated and seamless delivery of

TSPSC | Group-2 | Batch-3 | Admissions in progress | Call 7013495019 for admissions


citizen services by district administration through automation of workflow, backend computerization and data
digitization across participating departments. e-district is one of the 44 mission mode projects under National
e-Governance Plan (NeGP).

E-governance can also empower citizens by giving them a platform to participate in the decision-making
process, provide feedback, and hold the government accountable. It can help to bridge the digital divide and
bring about inclusive development.

Recently, the use of technology in the Election Commission of India especially mobile applications such as
‘cVIGIL’ and ‘Voter Helpline App’ have brought laurels to the commission.

The National e-Governance Plan (NeGP), takes a holistic view of e-Governance initiatives across the country,
integrating them into a collective vision, a shared cause. Around this idea, a massive countrywide
infrastructure reaching down to the remotest of villages is evolving, and large-scale digitization of records is
taking place to enable easy, reliable access over the internet. The ultimate objective is to bring public services
closer home to citizens, as articulated in the Vision Statement of NeGP.

Therefore, e-governance is not just a technological solution but a holistic approach to achieving good
governance in India. It requires the integration of technology, institutional capacity building, and a citizen-
centric approach to transform the governance process and deliver services that are responsive to citizens'
needs.

3. Analyze the role of the Bulwarks of Indian Constitution in ensuring the Good Governance in Indian
Democracy.

Approach:

1. Identify the keyword “Analyse”


2. Introduction: Various Bulwarks-Significance
3. Body: Role of Bulwarks in promoting good governance
4. Conclusion: Balanced

Model Answer:

The Constitution of India provides for various bulwarks that are essential to ensuring good governance in
Indian democracy. These bulwarks, which include the Judiciary, the Election Commission, and the Comptroller
and Auditor General (CAG), play a critical role in upholding the rule of law, ensuring transparency and
accountability, and protecting the rights of citizens.

Judiciary:

The Judiciary is an independent and impartial institution that plays a vital role in ensuring good governance in
India. The Constitution provides for an independent judiciary with the power to interpret the Constitution and
uphold the rule of law (Article-13 & 226). The Judiciary has the power of judicial review, which allows it to
strike down any law or executive action that is inconsistent with the Constitution. The Judiciary also has the
power to protect the fundamental rights of citizens, ensuring that they are not violated by the government or
any other entity.

It is participatory, accountable, transparent, efficient, and effective and follows the rule of law. It assures that
corruption is minimized, and the voices of the vulnerable are heard in decision-making. It is also responsive to
the present and the future needs of society. So, to achieve all these constitutional framers has appointed
higher judiciary as custodian of the constitution.

TSPSC | Group-2 | Batch-3 | Admissions in progress | Call 7013495019 for admissions


Election Commission:

The Election Commission is responsible for conducting free and fair elections in India. The Commission ensures
that elections are conducted in a transparent manner, and every citizen has an equal opportunity to
participate in the electoral process (Article 324-329). The Election Commission also ensures that political
parties and candidates follow the rules and regulations laid down by the Constitution and the law.

India being a democracy, it is important to give voice to the people and a chance to elect their leaders. In this
respect, election commission allows people to participate in the decision making process and promotes good
governance.

Comptroller and Auditor General (CAG):

The CAG is responsible for auditing the accounts of the government and ensuring that public funds are utilized
in a transparent and accountable manner (Article-148). The CAG provides an independent assessment of the
government's financial transactions, ensuring that public money is not misused or wasted.

All these bulwarks are enshrined in the Constitution of India and derive their constitutional authority from it.
Their primary objective is to ensure good governance in Indian democracy by upholding the rule of law,
ensuring transparency and accountability, and protecting the rights of citizens. They act as a check on the
power of the government and ensure that the government functions in a manner that is in line with the
principles of democracy and the Constitution.

The bulwarks of the Indian Constitution play a crucial role in ensuring good governance in Indian democracy.
They provide an institutional framework for upholding the rule of law, protecting the rights of citizens, and
ensuring transparency and accountability in the governance process. Their constitutional authority, roles, and
objectives are critical to the functioning of Indian democracy and the protection of citizens' rights and
interests.

4. Appropriate utilization of public fund and challenges of corruption are burning issues at present in public
administration. Critically examine

Approach:

1. Identify the keyword “Critically Analyse”


2. Introduction
3. Body
4. Conclusion

Model Answer:

Public administration is a branch of political science that covers the administrative services needed to help
build and strengthen society. It is traditionally used to define the formal arrangements under which public
organizations serve a government, ostensibly in the public interest, and was influenced primarily by Weber's
theory of bureaucracy, Northcote and Trevelyn’s recommendations. The appropriate utilization of public funds
and the challenge of corruption are indeed burning issues in public administration in India.

1. Appropriate Utilization of Public Funds:

The appropriate utilization of public funds is critical to ensure good governance in India. However, there
are several challenges in achieving this goal. Some of the major challenges include:

 Lack of transparency and accountability in the allocation and utilization of public funds.
 Insufficient monitoring and evaluation of public projects and programs.

TSPSC | Group-2 | Batch-3 | Admissions in progress | Call 7013495019 for admissions


 Inadequate institutional capacity and skill development in public administration.
 Political interference and bureaucratic red tape that hinder the efficient utilization of public funds.

To address these challenges, the government must ensure greater transparency and accountability in the
allocation and utilization of public funds. The government must also invest in building institutional capacity
and skill development in public administration, encourage citizen participation in the monitoring and
evaluation of public projects and programs, and reduce bureaucratic red tape and political interference.

2. Challenge of Corruption:

Corruption is a major challenge in public administration in India. Corruption undermines the principles of
good governance, transparency, and accountability. It also undermines public trust in the government and
hinders economic development. Some of the major challenges in combating corruption include:

 Weak anti-corruption laws and inadequate enforcement mechanisms.


 Political interference and influence in the functioning of anti-corruption agencies.
 A culture of impunity, where corrupt officials are not held accountable for their actions.
 Lack of public awareness and engagement in combating corruption.

To address these challenges, the government must strengthen anti-corruption laws and enforcement
mechanisms. The government must also ensure the independence and autonomy of anti-corruption
agencies, promote a culture of transparency and accountability, and encourage public engagement and
awareness in combating corruption.

In conclusion, the appropriate utilization of public funds and the challenge of corruption are critical issues
in public administration in India. The government must address these issues by promoting greater
transparency and accountability, investing in institutional capacity and skill development, reducing
bureaucratic red tape and political interference, strengthening anti-corruption laws and enforcement
mechanisms, and promoting public engagement and awareness.

5. A. Analyse the role of SERP and MEPMA in ensuring the effective implementation of schemes at the
grassroots level.
Approach
1. Introduce with the Poverty in India with a report
2. Role of SERP and MEPMA
3. Progress of SERP and MEPMA in Telangana
4. Conclusion: Strengthening these institutions for more effectiveness
Model Answer:
Global Multidimensional Poverty Index 2022 shows lack of access to nutrition, fuel, housing and sanitation
that keeps millions in poverty. India has uplifted 415 million people from multidimensional poverty between
2005-2006 and 2019-2021, according to the latest Global Multidimensional Poverty Index 2022 released by
the United Nations Development Programme (UNDP) and research centre Oxford Poverty and Human
Development Initiative (OPHDI). But India still has nearly 229 million multidimensional poor, in absolute terms
— the highest in the world. Of these, 90 per cent live in rural areas and the rest in urban areas.
Elimination of rural poverty has been a significant challenge for India since its independence. The
government has introduced various schemes to eradicate poverty, improve rural infrastructure, and provide
a better standard of living to people living in rural areas. However, the effective implementation of these
schemes at the grassroots level is often a challenge. In this context, the role of society and MEPMA becomes
crucial in ensuring the successful implementation of these schemes.
Society for Elimination and Reduction of Poverty (SERP)

TSPSC | Group-2 | Batch-3 | Admissions in progress | Call 7013495019 for admissions


 SERP is a sensitive and supportive structure of the Government to facilitate social mobilisation of the
poorest of poor in the rural areas of the state, except in the urban district of Hyderabad.
 It works for the promotion and strengthening of self-managed and financially sustainable community-
based organisations that are owned and controlled by the poorest of the poor.
 On the supply side, SERP has given a special focus on 100% coverage of marginalised communities.
 On the demand side of the problem, it works by building and nurturing Self Help Groups (SHGs) of women
and their federations.
 SERP works on a comprehensive multi-dimensional poverty alleviation strategy by focusing on
 Building strong / sustainable institutions for the poor,
 Financial Access - Leveraging resources through commercial banks, Livelihood value chain - sustainable
and diversified livelihoods,
 Human Development value chain, Accessing social safety nets and entitlements.

Progress of SERP in Telangana:


 The State Government in association with the Mandal Samakhyas of SERP, and other Town Level
Federations have created the Stree Nidhi Credit Cooperative Federation Ltd. (Stree Nidhi) as an apex
cooperative society for financing SHGs.
 SERP has initiated Gender sensitisation activities to the SHGs on various gender-related issues such as
eradication of dowry harassment, woman trafficking, child labour, child marriages and redressal of family
disputes etc
 Community Managed Rehabilitation Services (CMRS) is one of the critical components of Government
interventions through SERP, focusing on providing Rehabilitation Services and enhancing livelihood,
functioning, and community-integration of Persons with Disabilities and their families.
 Nearly 2,500 families of persons with disabilities in rural areas are getting rehabilitation services
through Neighbourhood centres in 74 mandals of 26 districts
 At present there are 45,60,518 SHG members in 4,35,364 SHGs organized into 18,397 Village Organizations
(VOs), 554 Mandal Samakhyas (MSs) and 30 Zilla Samakhyas (ZSs).
 This network of Community Based Organizations (CBOs) helped leverage and promote financial inclusion,
which was then expanded to include land based livelihoods, skills training, livelihoods support, access to
Government Programmes and initiatives to improve health and education outcomes.
MEPMA (Mission for Elimination of Poverty in Municipal Area)
 MEPMA (Mission for Elimination of Poverty in Municipal Areas) is a program launched by the Government
of Telangana to address poverty in urban areas.
 The program aims to provide sustainable livelihoods to the urban poor and improve their living conditions.
 MEPMA can also play an important role in ensuring the effective implementation of schemes at the
grassroots level.
 MEPMA can act as an intermediary between the government and the beneficiaries.
 The program can identify the needs of the beneficiaries and help the government design and implement
schemes that address their needs.
 MEPMA can also monitor the implementation of schemes to ensure that they reach the intended
beneficiaries and are implemented effectively.
Impact on economy:
 The economic dimension of a Mission for the Elimination of Poverty in Municipal Areas involves creating
economic opportunities for those living in poverty.
 This can be achieved by providing vocational training, microfinance, and job creation programs.

TSPSC | Group-2 | Batch-3 | Admissions in progress | Call 7013495019 for admissions


 By equipping individuals with the necessary skills and resources, they can start their own businesses
or find gainful employment, lifting themselves out of poverty.
Society: The social dimension of a Mission for the Elimination of Poverty in Municipal Areas involves
addressing the social causes of poverty.
 This includes providing education and health programs, community development initiatives, and social
safety nets.
 By addressing the root causes of poverty, these programs can help create a more equitable and just
society.
Impact on environment: Mission for the Elimination of Poverty in Municipal Areas recognizes the link between
poverty and environmental degradation.
 By implementing green infrastructure projects and renewable energy programs, the Mission can promote
environmental sustainability, which ensures that future generations have access to the resources they
need to thrive.
Progress of MEPMA in Telangana:
 “MEPMA (Mission for Elimination of Poverty in Municipal Areas)” is the Nodal Agency for implementing
Poverty Alleviation Programs in all urban areas of the State.
 Till date, 15,544 microenterprises have been established through urban SHG women with a total cost of
Rs.138.96 Crore under the Self-Employment Program (SEP).
 In 2022- 23 (till January 2023) an amount of Rs.114.35 Crore to 1,134 beneficiaries under the Self-
Employment Program (SEP). Since 2008 an amount of Rs. 15,144.43 crore is provided to 1.59 lakh
beneficiaries, in 2022-23 (till January 2023) an amount of Rs. 1,458.97 crore is provided to 18,680 SHGs
under Self Help Group - Bank Linkage (SHG-BL).
Conclusion:
The effective implementation of schemes at the grassroots level is critical for the elimination of rural poverty.
SERP and MEPMA can play important roles in ensuring that the schemes are implemented effectively and
reach the intended beneficiaries. Government can create awareness about the schemes, identify the needs of
the people, monitor the implementation of schemes, and provide financial and other forms of support.
5. B. The 73rd and 74th constitutional amendments 1992 to the Indian Constitution increases the effectiveness
of administration. Comment
Approach
Introduction: Enactment of the amendments- details.
Body:
1. A little background for democratic decentralization, how it reduced pressure off the Union & State list,
What more can be devolved, whether it increases efficiency in administration
Conclusion: Based on the substantiation given above, conclude how it increases efficiency with aspects like
timely delivery, people’s participation
Model Answer:
The 73rd Amendment 1992 added a new Part IX to the constitution titled “The Panchayats” covering provisions
from Article 243 to 243(O); and a new Eleventh Schedule covering 29 subjects within the functions of the
Panchayats. This amendment implements the article 40 of the DPSP, which says, “State shall take steps to
organise village panchayats and endow them with such powers and authority as may be necessary
The Constitution (Seventy Fourth Amendment) Act, 1992 has introduced a new part namely, Part IXA in the
Constitution, which deals with the issues relating to municipalities. The Constitution Amendment Act came
into force on 1st June, 1993.
Democratic Decentralisation and its background in India:
 Democratic decentralization is the process of devolving authority, functions and resources from the Centre
to the elected representatives at the lower levels so as to facilitate direct participation of the citizens in

TSPSC | Group-2 | Batch-3 | Admissions in progress | Call 7013495019 for admissions


governance. It requires the setting up of local level institutions to enable ordinary citizens at grass root
levels to participate in the decision-making, implementation, monitoring and sharing of the benefits of the
government activities.
 During India's freedom struggle, Mahatma Gandhi pleaded for 'Gram Swaraj' and effective
decentralization. Gandhian ideal of 'Gram Swaraj' was given due importance in the Directive Principles of
State Policy (Article 40)
 The Union Government also took many steps in regard to Panchayati Raj Institutions. In 1952, a
Community Development Programme was launched. In 1953, the National Extension Services Scheme was
introduced. After that, many committees were appointed to strengthen Panchayat Raj Institutions.
 The Government of India constituted committees like Balwant Rai Gopal Mehta Committee, Ashok Mehta
Committee, Dantwala Committee etc to provide recommendations such as three tier structure of
Panchayat and provision of adequate financial resources to Rural Local Governments.
 In 1978, the Janata Party government at the national level introduced the concept of Panchayati Raj or
decentralized governance through village councils. However, this initiative did not lead to significant
changes on the ground.
 It was only in the 1990s that the concept of democratic decentralization gained momentum in India.
 The 73rd and 74th Constitutional Amendments of 1992 were landmark changes that provided a
constitutional framework for democratic decentralization in India.
 The 11th Schedule contains the powers, authority and responsibilities of Panchayats.
 The 12th Schedule contains the powers, authority and responsibilities of Municipalities.
How Democratic Decentralisation reduced the pressure of the Union & State list
 The 73rd and 74th Constitutional Amendments of 1992 were landmark changes that provided a
constitutional framework for democratic decentralization in India.
 The 73rd Amendment Act, 1992, has added Eleventh Schedule, which enlists 29 subjects on which the
Panchayats have full authority
 Agriculture, Land improvement, Minor irrigation, Animal Husbandry etc
 The 74rd Amendment Act, 1992, has added Twelfth Schedule, which enlists 29 subjects on which the
Municipalities to have full authority
 Urban planning including town planning, Roads and bridge, Water supply for domestic, industrial and
commercial purposes etc
Administrative decentralisation:
 Democratic decentralization has reduced the administrative burden on the central government by
transferring powers and responsibilities to local self-government institutions.
 For instance, local governments are now responsible for maintaining public services such as water
supply, sanitation, health, and education.
 Article 243G of the Indian Constitution specifies the powers and responsibilities of Panchayats (local
self-government institutions) in rural areas.
 Article 243W of the Indian Constitution specifies the powers and responsibilities of Municipalities
(local self-government institutions) in urban areas.
Financial decentralisation:
 Local governments can now collect taxes and levies, which can be used to fund local development projects.
This has reduced the dependence of local governments on the central government for funding and has
enabled the central government to focus on larger-scale projects.
 Article 243H of the Indian Constitution allows Panchayats to levy taxes, duties, tolls, and fees, while Article
243X allows Municipalities to levy taxes on properties, professions, trades, and callings.
Political decentralisation:

TSPSC | Group-2 | Batch-3 | Admissions in progress | Call 7013495019 for admissions


 Local governments can now make decisions that are best suited to the needs of their communities, without
interference from the central government.
 Article 243K and 243Y provides the State Election Commission to conduct elections to local bodies in the
state in a free and fair manner.
 The Article 243S authorizes the state legislature to make legislation of ward committees in Municipalities.
Environmental protection:
 Municipal Corporations are responsible for solid waste management, controlling pollution, management
and conservation of natural resources such as forests, water bodies, and wetlands within their respective
jurisdictions., urban planning and development.
 The government has introduced the Solid Waste Management Rules, 2016
 The Delhi Development Authority has developed the Master Plan of Delhi, which emphasizes the
importance of sustainable development.
 The Pune Municipal Corporation has implemented a Rainwater Harvesting Scheme to recharge
groundwater levels.
Impediments of Democratic Decentralisation:
 Lack of financial resources
 Political interference
 Inadequate participation
 Caste and gender bias
 Fragmentation of power
Recent current context:
 The DMK government has devolved more funds and powers to the local bodies, demonstrating that
decentralisation held the key to speedy execution of work in a democracy
 A large number of urban local bodies (ULBs) of Telangana like Adibatla, Bhoothpur, Chandur, Chityala,
Gajwel, Ghatkesar have bagged the Swachh Sarvekshan 2022 awards based on their performance in
improving sanitation by dealing with the garbage disposal problem efficiently and effectively.
Conclusion:
In conclusion, democratic decentralization can increase efficiency in administration by promoting timely
delivery, encouraging people's participation, and improving resource utilization. However, it requires careful
planning, capacity building, and effective governance to address the challenges faced by local institutions.
When implemented effectively, democratic decentralization can bring about positive changes in local
governance and contribute to the overall development of the country.
6. A. Do you think the executive's responsibility to the legislature is required for effective implementation of
the principles of the Constitution? Justify
Approach
Introduction: What is the constitutional principle of individual and collective responsibility of executive
Body:
1. Why the responsibility of executive is given to legislature (Parliamentary supremacy). What is the status?
What happens if not given? Compared with the Presidential form of government.
Conclusion: Justify constitutionalism – responsibility to legislature
Model answer:
Executive is the important branch of government responsible for the implementation of laws and
policies adopted by the legislature. The official’s designations of the executive vary from country to country.
Some countries have presidents, while others have chancellors. The executive branch is not just about the
presidents, Prime Ministers. It also extends to the administrative hierarchy consisting of civil servants, which
is also known as permanent executive, which works under the supervision and control of Political executive.

TSPSC | Group-2 | Batch-3 | Admissions in progress | Call 7013495019 for admissions


How President is responsible to Legislature
 Addressing the Parliament: The President addresses the Parliament at the beginning of the first session of
each year, known as the Budget Session. In this address, the President highlights the government's policy
priorities and outlines the legislative agenda for the upcoming session.
 Signing Bills into Law: The President has the power to sign bills passed by both houses of Parliament into
law. However, the President can also send a bill back to Parliament for reconsideration if they have any
objections or concerns about its constitutionality.
 Summoning and Proroguing Parliament: The President has the power to summon both houses of
Parliament and to prorogue or dissolve them. This power is usually exercised on the advice of the Council
of Ministers.
 Seeking Information from the Prime Minister: The President can seek information from the Prime Minister
regarding any matter relating to the administration of the affairs of the Union, and the Prime Minister is
obliged to furnish such information.
How Prime Minister is responsible to Legislature:
 Tabling of Reports: The Prime Minister is required to table various reports, such as the Annual Report of
the Planning Commission, before the Parliament.
 Approval of Bills: The Prime Minister is responsible for introducing important bills and policies related to
the government's functioning, and getting them passed by the Parliament.
 The Prime Minister has to take the Parliament's approval for important bills related to finance and
taxation, and other key policy decisions.
 Vote of Confidence: The Prime Minister has to prove their majority in the Parliament through a vote of
confidence or no confidence.
 This ensures that the Prime Minister enjoys the trust and confidence of the Parliament and has the
necessary support to govern effectively.
How Council of Ministers are responsible to Legislature
 Collective Responsibility: The Council of Ministers functions as a collective body and is collectively
responsible to the Lok Sabha (the Lower House of Parliament) under Article 75 of the Constitution.
 This means that the Council of Ministers is collectively accountable to the Lok Sabha for all its policies,
decisions, and actions.
 Question Hour: The Members of Parliament (MPs) have the right to ask questions to the Council of
Ministers during the Question Hour in the Lok Sabha and the Rajya Sabha (the Upper House of Parliament).
The Council of Ministers is required to answer these questions and provide information related to the
government's policies, decisions, and actions
 Zero Hour: The Council of Ministers is also required to respond to the issues raised during the Zero Hour
in the Parliament, which is an informal time when MPs can raise issues of public importance
 Parliamentary Committees: The Council of Ministers is also required to appear before the various
Parliamentary Committees to provide information and clarification on various issues related to the
government's functioning
How Cabinet is responsible to the Legislature
 In India, the Cabinet, which is composed of the Prime Minister and the Council of Ministers, is collectively
responsible to the legislature for their actions and decisions. This principle is enshrined in the Constitution
of India and is known as the doctrine of collective responsibility
 The Lok Sabha has the power to ask questions of the Cabinet and hold it accountable for its policies and
actions.
 Members of the Lok Sabha can also introduce a vote of no confidence against the Cabinet, which, if passed,
can force the Cabinet to resign.

TSPSC | Group-2 | Batch-3 | Admissions in progress | Call 7013495019 for admissions


 Furthermore, the Cabinet is responsible for ensuring that the policies and decisions of the government are
in line with the laws and regulations of the country.
 The Cabinet is also responsible for presenting the annual budget to the Lok Sabha and ensuring that the
members of the House approve it.
 The executive branch is also accountable to the Rajya Sabha, but the Lok Sabha has more power in matters
of confidence and budgetary decisions.
Why Executive should be responsible to the Legislature
Transparency and Openness:
 Accountability to the legislature ensures transparency and openness in the decision-making process.
 The executive is required to report to the legislature on its activities, policies, and expenditures, which
promotes openness and transparency in the government's functioning.
Checks and Balances:
 The legislature serves as a check on the powers of the executive.
 By holding the executive accountable, the legislature ensures that the executive does not abuse its power
and that the government operates within the limits set by law.
Public Trust:
 Accountability to the legislature helps build public trust in the government. When the government is
transparent and accountable, the public is more likely to trust the government and its institutions.
Efficiency and Effectiveness:
 Accountability to the legislature promotes efficiency and effectiveness in government operations. When
the executive is held accountable, it is more likely to implement policies and programs effectively and
efficiently.
Public Participation:
 Accountability to the legislature ensures public participation in the decision-making process. By reporting
to the legislature, the executive ensures that the public is informed about the government's activities and
can participate in the democratic process.
Protection of Individual Rights:
 Responsibility to the legislature ensures that the government operates within the limits set by law,
including the protection of individual rights.
 By holding the government accountable, the legislature can prevent the executive from infringing on
individual rights and liberties.
Others:
 This control also ensures that the executive abides by the laws and prevents any kind of deviation from
the established laws.
 The control over the executive prevents the domination by a personality cult and provides a check on the
concentration of power into a single source
What happens if the Executive is not accountable to Legislature?
 The lack of accountability can lead to a lack of transparency and openness in the decision-making process.
 This can result in policies and decisions that are not in the public interest and do not reflect the needs
and concerns of the people.
 It can lead to a lack of efficiency and effectiveness in the implementation of government policies and
programs.
 This can result in a waste of public resources and a failure to address important issues such as poverty,
health, education, and infrastructure development.
 Thirdly, the lack of accountability can lead to a lack of trust in the government and the democratic process.

TSPSC | Group-2 | Batch-3 | Admissions in progress | Call 7013495019 for admissions


 This can result in public protests, demonstrations, and social unrest, which can further destabilize the
country.
 Fourthly, the lack of accountability can lead to a concentration of power in the hands of a few individuals
or groups, which can lead to corruption, nepotism, and abuse of power.
Conclusion:
Overall, responsibility to the legislature is a key component of constitutionalism. It promotes the separation
of powers, democracy, transparency, legitimacy, and the protection of individual rights. By ensuring that the
government operates within the limits set by law, responsibility to the legislature helps to create a system of
government that is fair, just, and accountable to the people.
6. B. Discuss various facets of Centre-State relations in ensuring good governance in Indian Democracy.
Approach
Introduction: Prime Minister Statement on Federalism, Any current issue of tussle or M K Stalin statement on
ISC
Body:
1. Necessity of Centre-State Relations
2. Legislative, Administrative, Financial relations between centre and state.
Conclusion: Conclude how good centre-state relations ensure good governance
Model Answer:
The Prime Minister of India Narendra Modi stated that Federalism is no longer the fault line of Centre-
State relations but the definition of a new partnership of Team India, which means federal India is no longer a
hindrance for better centre-state relations but rather it has given new hope to the mixed efforts of the Centre
and states which may be referred to as cooperative federalism.
Centre-State Relations and why it is necessary:
 Centre-state relations refer to the relationship between the central government and the state
governments in a federal system of governance. In India, the Constitution provides for a federal system of
governance where the powers and responsibilities are divided between the central and state
governments.
 Part XI of the Indian Constitution specifically deals with centre-state relations. It has been bifurcated
into legislative and administrative relations.
 Further, in Part XII, provisions related to financial relations are laid down.
Legislative Relations (Article 245-255)
 Articles 245 to 255 in Chapter I, Part XI of the Constitution make an elaborate division of legislative powers
between the Union and the States with regard to
 territorial jurisdiction
 subjects/items for law-making.
 They are enumerated in three lists known as Union List, State List and the Concurrent List. The Centre and
the States are required to enact and legislate within their sphere of activity.
 Parliament can make laws on any matter in the State List for implementing the international treaties
agreements or conventions. For e.g., United Nations (Privileges and Immunities) Act, 1947; Geneva
Convention Act, 1960; Anti-Hijacking Act, 1982 and certain laws relating to the environment.
 According to Article 249, Parliament can make law on any subjects. If the Rajya Sabha declares that such
a subject to be a subject of national importance. However, such a resolution must be supported by two-
thirds of the members present and voting.
 Parliament can enact a law to constitute the Public Service Commission for the concerned State, if a
resolution to that effect is passed by the Legislature of that State. Once this law is passed, the power of
amendment and repeal of the law rests with the Parliament but not with the State (Art 252).

TSPSC | Group-2 | Batch-3 | Admissions in progress | Call 7013495019 for admissions


 The Constitution also makes provision for the creation of a new All-India Service by the Parliament on the
recommendation of the Rajya Sabha.
 Parliament has the power to legislate for the Union Territories.
 The Parliament alone can make 'extraterritorial legislation'. Thus, the laws of the Parliament are also
applicable to the Indian citizens and their property in any part of the world.
Parliament Control over State List
The Constitution empowers the Parliament to exercise control over the State's legislative matters in the
following ways:
 Article 200 of the Constitution provides that certain types of bills passed by the State legislature may be
reserved by the Governor for the consideration of the President.
 Bills on certain matters enumerated in State List can be introduced in the State legislature only with prior
sanction of the President. For example, the bills imposing restrictions on the freedom of trade and
 The President can direct the States to reserve money bills and other financial bills passed by the State
legislature for consideration during a financial emergency.
Administrative Relations (Article 256-263):
 Chapter II of Part XI from Article 256 to 263 deals with e administrative relations between the Union and
the States. As in legislative relations, the Centre has more power than the States in administrative relations
too. The executive power (of Centre and States) is coextensive with legislative power, which means:
 the Centre can execute all matters on which Parliament can make laws.
 The Centre's executive power extends to the whole of India. List.
 The State can execute all matters enumerated in the State
 The State executive power is to be exercised within its territory.
 Under Article 256, the State executive power has to be exercised in a way so as to ensure compliance with
the laws made by the Parliament.
 Under Article 257 (1) of the Constitution, the Executive power of every state shall be exercised as not to
impede or prejudice the exercise of the executive power of the Union.". This means, powers of the Centre
are superior to the executive powers of the States.
 A State can also, with the consent of Union government, confer administrative functions on it
 The Centre can exercise rights, authority and jurisdiction conferred on it by any international agreement.
Extent of Union Executive Power:
The President may, with the consent of the State, entrust either conditionally or matter to which the executive
power of the Union extends, such as the following:
 The Union government can ask the States to create and maintain the communications, related to national
and military importance.
 Union government declares National Highways and Waterways, maintenance of which is the duty of the
states
 The Union government can give directions to the States for the protection of Railways, maintenance of
armed forces, and Union government's property.
 The Union government has control over States through All-India Services, common to both the Centre and
States. However, their appointment, service conditions, and taking disciplinary action against them can
only be done by the Union Government.
Financial Relations (Article 268-293):
 Both the Parliament and State legislature can levy taxes on subjects enumerated in the Concurrent List.
 Taxes that have an inter-State base are levied by Centre and those with a local base by the State.
 The residuary power of taxation on subjects not enumerated in any of the three lists is vested in the
Parliament. Under this provision, the Parliament imposes gift tax, wealth tax and expenditure tax.

TSPSC | Group-2 | Batch-3 | Admissions in progress | Call 7013495019 for admissions


Tax Revenue Distribution
Grant-in-Aid and Loans:
 Article 275, the Centre has to provide grants-in-aid to the States, for both general purpose and specific
purpose.
 However, as per the recommendations of the Finance Commission, the Parliament decides to which States
need grant assistance and to what extent. What are the challenges faced by NGOs in India:
Borrowing power of Centre and States:
 Article 292 and 293, of the constitution mention the borrowing power of Union and State governments.
 The State governments can borrow money only within the territory of India upon the security of the
Consolidated Fund of the state, with the consent of the Central Government.
 Article 280, the Presidents appoints a Finance Commission which makes recommendations for tax revenue
distribution between Union and States.
Emergency Provisions and Centre-State Relations:
 During an Emergency (under Article 352), all powers are centralized in the Union government. Union
executive power extends to giving directions to States in the exercise of their executive powers
 When a State Emergency (under Article 356) popularly known as President's Rule is imposed, the entire
control of the State administrative machinery comes under the Union. The President can assume all or any
of the functions of the State government.
 During a Financial Emergency (under Article 360), the Union can give directions to any State to observe
canons of financial propriety.
 In short, during an Emergency, the Union can withdraw all powers from the State, also the financial
support it gives, and function as a Unitary State.
Conclusion:
In conclusion, Centre-State relations are essential in ensuring good governance in Indian democracy. Effective
coordination, decentralization of power, sharing of resources, and checks and balances between different
levels of government can help in providing efficient and effective governance to the citizens of India.
7. A. Critically examine the implementation of MGNREGA program in the alleviation of rural poverty.

Approach:

1. Identify the keyword “Critically Examine”


2. Introduction: Recent context-Allocation of Budget for MGNREGA
3. Body:
a. Brief about MGREGA
b. Positives-how it alleviates rural poor
c. More Focus on Criticism-cuts in budget
d. Suggestions or way forward
4. Conclusion: What should be done/Balanced/futuristic based

Model Answer:

The Standing Committee on Rural Development, which submitted its report in the Rajya Sabha, said the budget
estimates for MGNREGS have been reduced by ₹ 29,400 crore for 2023-24 when compared to the revised
estimates of 2022-23. However, according to the finance Minister, the fund allocation for Mahatma Gandhi
National Rural Employment Guarantee Scheme (MGNREGA) has not gone down. As it is a demand driven
scheme and as and when the demand increases, the Centre provides money.

About MGREGA:

TSPSC | Group-2 | Batch-3 | Admissions in progress | Call 7013495019 for admissions


Mahatma Gandhi National Rural Employment Guarantee Scheme (Mahatma Gandhi NREGS) is a
demand driven wage employment programme which provide for the enhancement of livelihood security of
the households in rural areas of the country by providing at least one hundred days of guaranteed wage
employment in every financial year to every household whose adult members volunteer to do unskilled
manual work. One of the goals of MGNREGA is to enhance livelihood security of the rural poor through
generation of wage employment opportunities in works leading to creation of durable assets

The Centre has cut the allocation for the Mahatma Gandhi National Rural Employment Guarantee Scheme
(MGNREGS) by 21.66 percent for 2023-24. Finance Minister has allocated Rs.60,000 crore for the scheme,
lower than the budgetary estimate of Rs 73,000 crore for 2022-23. The reduction is sharper when compared
with the revised estimate of Rs 89,400 crore for the current financial year.

How MGNREGA helped people?

 Provided guaranteed wage employment of 100 days in a financial year, thus enhanced social security and
living standards.
 Various research reveals that this programme has led to an increase in agriculture wage rates and boosts
the real daily agricultural wage rates. The wage effect is equal for both men and women and is in favour
of unskilled labour.
 Social audit increases transparency and trust among rural communities.
 Prevented seasonal or distressed migration towards cities and given work opportunity at native place and
increased household income which is used for food security, education of dependents, health care and
debt repayments. In some cases, earnings were utilized to acquire durable assets and created amenities
in households.
 Provided work opportunities for weaker sections of the society, i.e. Dalits and Adivasis. Various studies
revealed that MGNREGA has succeeded in high participation from marginalised groups including the SCs
and STs.

The role and importance of MGNREGA were visible during the corona pandemic times when it acted as a ray
of hope for the needy in times of distress

Challenges:

 The program is primarily focused on providing short-term employment opportunities to rural households,
which may not be sufficient to address the underlying causes of poverty, such as lack of access to
education, healthcare, and other basic services. Moreover, the program does not address the structural
issues in the rural economy, such as low productivity, low wages, and limited job opportunities in non-
farm sectors.
 The continuous pendency in the disbursement of the Centre's share of funds under wages and material
components of the scheme in respect of several states/Union Territories.
 According to the information received from the Department of Rural Development, it has been
observed that as on January 25, 2023, Rs 6,231 crore in wages and Rs 7,616 crore in material
component is pending liability on the Centre's part.
 A scheme of such huge proportion like MGNREGA which caters to the nook and corners of the country
and has millions of beneficiaries enrolled as job card holders, such delay in wage payments and material
fund release would only deter the needy person from availing the benefits under MGNREGA and would
also cause them further turpitude for non-receipt of money when the sole intent of the scheme was to
provide timely relief to poor in times of duress.
 There is an issue of unspent balance under MGNREGS during the last three years -- Rs 2,913.32 crore
(2019-20), Rs 5,270.76 crore (2020-21) and Rs 6,454.87 crore (2021-22).

TSPSC | Group-2 | Batch-3 | Admissions in progress | Call 7013495019 for admissions


 Also, the recent budget cuts have raised concerns from various places. It received the lowest allocation
in 17 years in the Union Budget 2023-24 released on February 1, 2023, apart from the first two years of
the scheme.
 Though the government sources have pointed out that expenditure on MGNREGS is need-based, and can
always be increased if such a requirement is felt. The Budget allocation, thus, does not mean no more
money can be spent on the rural employment scheme. Activists and worker unions claim that by cutting
down Budget allocation, the government sends out a notice that it wants to spend less on MGNREGS, and
hence fewer initiatives are undertaken under the scheme.
 Data shows that the scheme hasn’t been evenly implemented in terms of the wages given across all the
states.
 The scheme witnesses rampant corruption and the bureaucracy overlooks it. Since June 2014, the scheme
has slowed down progress.
 Due to low wage rates, many people are backing out of the scheme leading to middlemen taking control.
 There is also a need to enhance the skills of the workers under this scheme.

Suggestions/Way forward

 The Centre must allocate more funds in the budget since the dearth of funds could lead to halting the
work, thus causing the interruption. Also, the laborers must be paid well in time.
 The entire scheme could be decentralized so as to make the working a lot better.
 An increase in wages can help in making the scheme more successful.
 Also, the central government is now trying to include skill development in this scheme so as to increase
the skilled labor in rural areas.
 It should also study and understand why exactly the funds aren’t being utilized properly and take the steps
required so as to reach as many people as possible.

While the MGNREGA program has provided employment opportunities to millions of rural households in India,
its implementation has been marred by several challenges. Addressing these challenges is crucial for the
program to be effective in alleviating rural poverty in India.

7. B. Health is one of the focus areas of Telangana government in ensuring social justice. Substantiate

Approach:

1. Keyword “Substantiate”
2. Introduction: Significance based/allocation to health sector in recent budget
3. Body:
a. Initiatives taken by government
b. How it affected/their impact in improving health and ensuring social justice
4. Conclusion: Substantiation based-How health plays a significant role in social justice, or significance of
healthy population.

Model Answer:

Telangana has allocated 5% of its total expenditure towards health. One of the most important indicators of
human development in a community is improvement in health outcomes. Acknowledging the need for a
focused and dynamic policy health sector in the State, the Government of Telangana has formulated various
initiatives and schemes to address key health issues in the community.

Infrastructure:

TSPSC | Group-2 | Batch-3 | Admissions in progress | Call 7013495019 for admissions


 The Government runs a network of primary, secondary, and tertiary healthcare facilities in order to
provide universal health coverage and guarantee the best service delivery to patients throughout the
State.
 The patients in the State are catered to through 4 Civil Dispensaries, 2 Eye Hospitals, 4 Maternity
Hospitals, 1 Child Hospital, 1 ENT Hospital, 1 Fever Hospital, 2 Chest Disease Hospitals, 33 District
Hospitals, 44 Sub District Hospitals, 31 General & Speciality Hospitals, 17 Medical colleges, 119
Community Health Centres, 20 Area Hospitals, 636 Rural Primary Health Centres, 232 Urban Primary
Health Centres, 4,745 Health Sub Centres, 2,250 Health and Wellness Centres, and 342 Basti
Dawakhanas.
 The Government sanctioned nine new Government Medical Colleges with a cumulative intake capacity of
900 MBBS students at Asifabad, Jayashankar Bhupalpally, Jangaon, Kamareddy, Karimnagar, Khammam,
Rajanna Siricilla, Vikarabad and Nirmal.
 In 2021, the Government laid the foundation stone and sanctioned Rs 1,100 crore for the construction of
a super specialty hospital in Warangal as part of its plan to develop Warangal into a health city and a
medical tourism destination.

Milestones:

 Telangana was the only State among the 19 larger States in the country to have demonstrated a strong
overall performance on NITI Aayog’s Annual Health Index 2019-20, ranking third.
 Telangana has received the best performance award in malaria elimination and progressed from Category
II to Category I from 2015 to 2021.
 Telangana has the highest coverage of the precautionary (booster) dose vaccination in the 18+ age group
among larger States.

Initiatives:

 ‘Basti Dawakhanas’ are an innovative initiative of the Government of Telangana to bring healthcare
services close to the urban poor. 342 Basti Dawakhanas have been established by the State in urban areas
so far, with one Basti Dawakhana serving 5,000 to 10,000 persons.
 These centres provide 53 different kinds of free health services including outpatient consultation,
medicines, basic lab diagnosis, antenatal/postnatal care, and screening for non-communicable
diseases. Specialist consultation is also provided at Basti Dawakhanas through Telemedicine.
 To ensure that people residing in rural areas have access to the same quality healthcare services as their
urban counterparts, including round-the-clock access to doctors, the Government has started the process
of converting sub-centres to “Palle Dawakhanas”.
 The Palle Dawakhanas will deliver comprehensive primary healthcare services to the local population,
will stock medicines for common ailments, and have a fully-equipped laboratory to conduct basic
diagnostic tests. Specialist consultations would be available through telemedicine facilities.
 Under Ayushman Bharat – Pradhan Mantri Jan Arogya Yojana (AB PMJAY)- Aarogyasri an additional 646
treatments were made available and applicable through the government hospitals in the State.
 Telangana has implemented a number of upgrading programmes for AYUSH dispensaries and hospitals,
making it the only State with pharmacies in all three AYUSH systems (Ayurveda, Unani, and Homeopathy).
 Government has launched Rs. 5/- per meal programme for attendants of patients in 18 major hospitals in
Hyderabad / Greater Hyderabad Municipal Corporation (GHMC).
 Key programmes to reduce communicable diseases like National Tuberculosis(TB) Elimination Programme,
National Vector Borne Disease Control Programme, HIV/AIDS and Telangana State AIDS Control Society
(TSACS), National Leprosy Elimination Programme have been implemented.

TSPSC | Group-2 | Batch-3 | Admissions in progress | Call 7013495019 for admissions


 Also programmes for reducing non-communicable diseases like National Programme for Prevention and
Control of Cancer, Diabetes, Cardiovascular Diseases and Stroke (NPCDCS), National Tobacco Control
Programme etc have been implemented.

“Ensuring healthy lives and promoting well-being for all at all ages” is one of the crucial Sustainable
Development Goals, which is of intrinsic importance. It can allow society to contribute more significantly and
effectively to economic development. Telangana’s strength lies in its strong public healthcare system.
Investments in the State’s health infrastructure have increased significantly during the past two years, both in
terms of size and quality. The Government should continue to work toward delivering high-quality healthcare
to all individuals with a specific focus on availability, accessibility, and affordability

8. A. Give an outline of Sugamya Bharat Abhiyan and assess the status of its implementation.
Approach:
1. Keyword “Give an Outline” and “Assess”
2. Introduction: aim of the program
3. Body
a. About the program
b. Targets
c. How far it was implemented or successful
4. Conclusion: Way forward
Model Answer:
Department of Empowerment of Persons with Disabilities (DEPwD) launched Accessible India Campaign
(Sugamya Bharat Abhiyan) as a nation-wide Campaign for achieving universal accessibility for Persons with
Disabilities (PwDs) on December 3, 2015. It has three important verticals, namely - the Build Environment, the
transportation sector and the ICT ecosystem.
The Accessible India Campaign drew inspiration from united Nations Convention on Rights for Persons with
Disabilities (UNCRPD; 2007) to which India is a signatory. The Action Plan and targets of the Accessible India
Campaign have been derived from Goal 3 of the Incheon Strategy which endeavors to “Make the Right Real”.
The vision of Accessible India Campaign is to create a barrier free environment for independent, safe and
dignified living of Persons with Disabilities. The Vision statement declares: "Accessible India. Empowered
India."
To provide full legislative cover to the Campaign and Right to accessibility, the Government enacted the Rights
for Persons with Disabilities (RPwD) Act, 2016 which came into force from April 2017. Accessibility became a
Right for the Divyangjan unlike previously, when it was being seen merely as a welfare measure. Non-
compliance of the provisions of the Act or Rules thereunder has been made punishable by fines and
imprisonment. Thus, the Accessible India Campaign became an instrument to actualize the provisions of the
Act.
Targets:
Following are the targets under the three verticals of the campaign:
 Built Environment Accessibility
 Target 1.1: Conducting accessibility audit of at least 25-50 most important government buildings and
converting them into fully accessible buildings in the selected 50 cities.
 Target 1.2 : Converting 50% of all the government buildings of National Capital and all the State
capitals into fully accessible buildings
 Target 1.3: Conducting audit of 50% of government buildings and converting them into fully accessible
buildings in 10 most important cities / towns of all the States (other than those, which are already
covered in Target 1.1 and 1.2 above)
 Transportation System Airports

TSPSC | Group-2 | Batch-3 | Admissions in progress | Call 7013495019 for admissions


 Target 2.1: Conducting accessibility audit of all the international airports and converting them into
fully accessible international airports
 Target 2.2: Conducting accessibility audit of all the domestic airports and converting them into fully
accessible airports Railways
 Target 3.1: Ensuring that A1,A & B categories of railway stations in the country are converted into fully
accessible railway stations
 Target 3.2: Ensuring that 50% of railway stations in the country are converted into fully accessible
railway stations Public Transport (Buses)
 Target 4.1: Ensuring that 25% of Government-owned public transport carriers in the country are
converted into fully accessible carriers
 ICT Eco- System
 Target 5.1: Conducting accessibility audit of 50% of all government (both Central and State
Governments) websites and converting them into fully accessible websites
 Target 5.2: Ensuring that at least 50% of all public documents issued by the Central Government and
the State Governments meet accessibility standards.

Initially, targets were set under the Sugamya Bharat Abhiyan to be completed by 2016-207, but several targets
were missed by one to three years. Thus, the government set March 2020 as the new deadline for completing
the goals under this campaign. Since the government missed the target in 2020, the targets timeline has been
further extended to 2022.

Achievements so far:
Built up environment:
 Access Audit of 1671 buildings completed.
 1607 Government buildings, including 1030 Central Government building have been provided with
features of accessibility.
Transportation sector: Efforts are being made to make the transportation sector accessible.
 Airports: 35 International Airports and 55 Domestic Airports have been provided with features of
accessibility. Ambulifts are available at 12 airports.
 Railways: All 709 A1, A & B category railway stations have been provided with seven Short Term Facilities.
603 railway stations have been provided with 2 Long Term Facilities.
 Roadways: 44,153 (30.01%) buses have been made partially accessible and 8,443 (5.73%) made fully
accessible
ICT Ecosystem:
 Websites: Nearly 603 Central and State/UT government websites have been made accessible.
 Accessibility in TV viewing:
 Being ensured in a phased manner.
 19 private news channels are telecasting partially accessible news bulletins.
 2447 news bulletins have been telecast with subtitling/sign-language interoperation.
 9 general entertainment channels have telecast 3686 scheduled programs/movies using subtitling.
Education:
 Out of 11,68,292 government and government aided schools, 8,33,703 schools (71%) have been made
barrier free with the provision of ramps, handrails and accessible toilets
Institutional Literacy Material
 The Department has developed an Easy Reckoner of 10 basic features of Accessibility for ease of
understanding.
 Volume 1 of the series of guidebooks for professionals titled ACCESS-The Photo Digest on Public Centric
Buildings was launched on 2nd March 2021.

TSPSC | Group-2 | Batch-3 | Admissions in progress | Call 7013495019 for admissions


 Volume 2 of the series of guidebooks for professionals titled ACCESS-The Photo Digest on Airports was
launched on 19.11.2021.
Monitoring:
 Activities under the Accessible India Campaign are being monitored through a Management Information
System (MIS) portal.
Formulation of Sector Specific Standards of Accessibility:
 The task of formulation of Sector Specific Standards/Guidelines of Accessibility is underway by respective
Ministries/Departments, including Civil Aviation, Roadways, Railways, School and Higher Education,
Culture, Tourism, MHA, Banking, Consumer Affairs and Sports.
 Department is lending support through sensitization, handholding and by recommending experts in field
of accessibility.
Sugamya Bharat App:
 Helping in crowdsourcing grievances of accessibility being faced on ground in infrastructure and services
and forwarding for redressal.
 Helpful also in creating sensitization and awareness generation about importance of accessibility.
 COVID-19 related complaints which are meant only for Divyangjans are being given top priorities now. To
be cleared within 3 days.
Despite the progress made, there are still challenges that need to be addressed. Some of these challenges
include:
 Enforcement: There is a need for better enforcement of accessibility standards. Many public spaces and
buildings do not meet the accessibility requirements, and there is a lack of monitoring and enforcement
mechanisms.
 Funding: The program requires more funding to achieve its objectives. The budgetary allocation for the
program needs to be increased to ensure that all public spaces and infrastructure are made accessible.
 Awareness: There is a need to raise awareness among people, including policymakers, about the
importance of accessibility. This will help in building support for the program and ensuring its success.
The Sugamya Bharat Abhiyaan has made progress in making public spaces and infrastructure accessible to
persons with disabilities. However, there are still challenges that need to be addressed, and more needs to be
done to ensure that the program achieves its objectives.
8. B. Critically examine various challenges in the implementation of Corporate Social Responsibility in India.
Approach
Introduction: What is Corporate Social Responsibility?
Body:
1. Need and implementation of CSR in India
2. Challenges faced by CSR
Conclusion: Positive outlook on how CSR can promote sustainable development
Model Answer:
Corporate Social Responsibility (CSR) refers to a company's commitment to operate in an economically,
socially, and environmentally sustainable manner. Under the Companies Act, 2013, certain classes of
profitable companies are required to spend at least 2 percent of their average net profit of the preceding three
financial years on CSR activities in a particular financial year. The Corporate Social Responsibility concept in
India is governed by Section 135 of the Companies Act, 2013 (‘Act’), Schedule VII of the Act and Companies
(CSR Policy) Rules, 2014.
Implementation of Corporate Social Responsibility in India:
 India is the first nation within the world which has made Corporate Social Responsibility (CSR) obligatory,
taking after change in April 2014, to The Company Act, 2013. Presently Businesses get openings to
contribute their benefits in zones such as education, destitution, sexual orientation balance, and hunger.

TSPSC | Group-2 | Batch-3 | Admissions in progress | Call 7013495019 for admissions


 Be it a private sector company or a public sector company, Corporate Social Responsibility CSR has to be
adhered to by all listed companies to spend 2% of their average net profits of three years on CSR. If a
company falls in either of the following criteria for compulsion, they need to form a CSR committee.
Companies:
 That has a net worth of ₹ 500 crores or more, or
 That have an annual turnover of ₹ 1000 crores or more, or
 That generate a profit of ₹. 5 crores or more
 In case a company fails to comply with the provisions relating to CSR spending, transferring and utilising
the unspent amount, the company will be punishable with a minimum fine of Rs 50,000 which may
increase to Rs 25 lakh.
 Further, every officer of such a company who defaults in compliance will be liable for a punishment which
is imprisonment for a term which may extend to three years or with a minimum fine of Rs 50,000 which
may increase to Rs 5 lakh, or with both.
Challenges faced by Corporate Social Responsibility:
 Corporate Social Responsibility (CSR) has become a significant aspect of business activities worldwide,
including in India. However, several challenges have hindered the effective implementation of CSR in the
country
Social Challenges:
 Major challenge is the lack of awareness and understanding of CSR among stakeholders, including
companies, consumers, and the government.
 Many companies are still not familiar with CSR practices, and some do not see the relevance of investing
in CSR activities
 Although private companies reveal data to MCA, which is showcased at the National CSR Portal, data
for numerous companies is either non-existent or contradictory.
 Another challenge is the diversity of India's social and cultural fabric, which makes it challenging to design
and implement CSR programs that cater to the needs of different communities.
 The lack of standardization in CSR activities also means that companies often implement CSR activities
that are not relevant to the local context, leading to ineffective CSR programs.
Economic challenges:
Resource Allocation:
 The allocation of resources is one of the most significant economic challenges of implementing CSR in
India. Many companies, especially SMEs, often struggle to allocate the necessary financial and human
resources to implement CSR initiatives effectively. This can be due to financial constraints, limited
expertise, or the lack of a dedicated CSR team.
Cost:
 Implementing CSR activities can be expensive, and the costs associated with it can be a significant
challenge for companies.
 The expenses can include direct costs such as financial contributions, employee time, and other indirect
costs such as training, monitoring, and evaluation of the CSR program.
Return on Investment:
 Many companies see CSR activities as an expense rather than an investment, leading to a lack of long-term
commitment to CSR activities.
 The city has a skilled and diverse talent pool of technology professionals, and its cost-effective business
environment and supportive government policies have attracted significant investment from both
domestic and international companies.
Political Challenges:
Regulatory Framework:

TSPSC | Group-2 | Batch-3 | Admissions in progress | Call 7013495019 for admissions


 There is a lack of a clear regulatory framework for CSR activities in India, making it difficult for companies
to comply with the law.
 While the Companies Act 2013 mandates that companies spend at least 2% of their average net profits in
the preceding three financial years on CSR, there are no clear guidelines on how to allocate these funds
effectively.
 This can lead to confusion and a lack of clarity on which CSR activities to focus on, making it difficult to
allocate resources effectively.
Lack of Monitoring:
 While companies are required to report their CSR activities in their annual reports, there is no independent
agency to monitor the implementation of CSR initiatives.
 This can lead to a lack of accountability and transparency, making it difficult to measure the impact of
CSR initiatives.
 In 2021, the Ministry of Corporate Affairs released a report stating that over 6,000 companies had not
spent their entire CSR budget for the year 2019-20
Corruption:
 There have been instances where companies have used CSR funds for personal gain, leading to a lack of
trust in the system. Corruption can also make it difficult to ensure that the funds allocated for CSR activities
are used effectively and efficiently.
 Coal Scam: In 2014, the CBI (Central Bureau of Investigation) booked several companies for allegedly
diverting CSR funds meant for the welfare of the people in the coal mining areas of Jharkhand and
Chhattisgarh.
 To avail CSR funds of Rs 33 lakhs for a CSR project, Karnataka Welfare Association for the Blind from
Bengaluru, paid close to Rs 14 lakh to a social activist as ‘consultation fees’ to avail the larger fund
reservoir from a Dubai-based company, Etisalat Software Solutions Pvt Ltd
Political Interference
 Political interference can also be a challenge in implementing CSR activities in India. There have been
instances where political leaders have used CSR funds for their own gain, leading to a lack of trust in the
system.
 In 2018, the Union Territory of Puducherry mandated that all government schools should display the
photograph of Chief Minister V Narayanasamy and Education Minister R Kamalakannan.
 The move was met with criticism, with many people claiming that it was a misuse of CSR funds, as the
funds were being used to buy the photographs and frames.
Conclusion:
CSR activities promote sustainable development by encouraging companies to adopt sustainable business
practices, reduce their environmental impact, and contribute to the social and economic development of local
communities. Overcoming the challenges faced by CSR in India requires a collaborative effort by all
stakeholders, with a focus on transparency, accountability, collaboration, capacity building, long-term
planning, and a robust regulatory framework.
9. A. SHGs remain one of the few successful initiatives in people centered participatory development.
Substantiate
Approach
Introduction: Report based
Body:
1. What are SHGs and need of them
2. Progress of SHGs in India
3. Limitation of SHGs
Conclusion: How to overcome the challenges

TSPSC | Group-2 | Batch-3 | Admissions in progress | Call 7013495019 for admissions


Model Answer:
The Economic Survey 2022-2023, presented by the Union finance minister to Parliament January 31
2023 has a special mention of India’s little-known but high-impact banking operation: the women-led self-help
groups (SHG) disbursing billions of rupees every year.
“The number of SHGs credit linked has grown at a CAGR of 10.8 percent during the last ten years (FY13 to
FY22), while credit disbursement per SHG has grown at a CAGR of 5.7 per cent during the same period,” notes
the Economic Survey 2022-2023. People-centred participatory development is an approach to development
that places people at the center of the development process.
What are Self-Help Groups
 Self Help Groups (SHGs) are community-based organizations that consist of women who come together
voluntarily to form a group for the purpose of mutual help and support.
 SHGs are a model of empowerment for women and have been successful in promoting financial inclusion,
social mobilization, and community development.
 SHGs are typically formed at the village or community level and consist of 10 to 20 women.
 The members of the SHG contribute a small amount of money each month to create a pool of savings,
which can be used for emergency needs or to provide loans to members.
 The SHGs also provide a platform for women to discuss and address social issues such as domestic
violence, health, and education.
How far SHGs are successful:
 India boasts of some 12 million SHGs, of which 88 per cent are all-women-member ones.
 In 1992, these groups were linked to banks for disbursal of small loans — called SHG Bank Linkage Project,
SHG-BLP — for taking up livelihood options, like livestock rearing or setting up a tailoring unit.
 By 2022, these groups operate a financing operation that is regarded as the world’s largest microfinance
project.
 According to data cited in the Economic Survey 2022-2023, SHG-BLP covers 142 million families with saving
deposits of Rs 47,240 crore.
 The loans under SHG-BLP, there are hardly any loans turning into bad ones that are to be waived off.
 Notably, SHGs’ bank repayment is more than 96 per cent, underscoring their credit discipline and
reliability,” says the Economic Survey 2022-2023
 Across the country, states have been declaring special economic packages for SHGs as part of overall
livelihood development programmes. Under the National Rural Livelihood Mission (NRLM), the
government has set a target of increasing each SHGs income to Rs 1 lakh by 2024. To do so, microfinance
linked to livelihood development is the key strategy.
Limitations of SHGs:
 The survival of many SHGs is dependent on the promotional agencies. SHGs are liable to collapse if these
agencies withdraw their funding.
 The majority of SHGs do not take advantage of new technology advancements and talents.
 The majority of SHGs are unregistered. They are governed on the basis of mutual trust among the
members. The SHG members' savings may not be safe, causing distrust among the members.
Conclusion:
In summary, to overcome the challenges faced by SHGs in India, it is essential to strengthen the capacity of
SHG members, facilitate access to finance, establish market linkages, address social and cultural barriers, and
establish effective monitoring and evaluation mechanisms. By doing so, SHGs can continue to play a vital role
in empowering women and promoting inclusive and sustainable development in India.
9. B. NGOs strive to improve social and political conditions in Indian Democracy. Critically analyse
Approach
Introduction: What is a NGO and their role in Civil Society?

TSPSC | Group-2 | Batch-3 | Admissions in progress | Call 7013495019 for admissions


Body:
1. How NGOs strive to improve social and political conditions with examples
2. Challenges faced by NGOs
Conclusion: Conclude with how to overcome the challenges
Model Answer:
NGOs, or non-governmental organizations, are an essential component of civil society in India. They
play a crucial role in advocating for social and political change and striving to improve the conditions of Indian
democracy. However, their impact can vary depending on the conditions and dimensions of their work.
NGOs play a crucial role in civil society by advocating for social, economic, and political change, providing
essential services, conducting research and analysis, monitoring and evaluating government policies and
programs, and building the capacity of communities and individuals. Some Non-Governmental Organisations
(NGOs) in India are CRY (Child Rights and You), Give India Foundation, Care India, Pratham etc
How NGOs strive to improve social conditions in India:
Education:
 Education is a fundamental right of every individual, but in India, there are still many children who do not
have access to education.
 NGOs like Pratham, Teach for India, and Akanksha Foundation work towards providing education to
underprivileged children by establishing schools and conducting after-school programs.
Women Empowerment:
 Women in India face significant social and economic challenges, such as limited access to education and
employment opportunities.
 NGOs like SEWA, Maitri, and Safety work towards empowering women by providing them with training,
access to credit, and organizing them into self-help groups.
Health and Nutrition:
 NGOs like Save the Children, Smile Foundation, and CRY work towards improving the health and nutrition
of children in India by providing healthcare facilities, nutrition programs, and awareness campaigns.
How NGOs strive to improve political conditions in India:
Electoral Reforms:
 NGOs like the Association for Democratic Reforms (ADR) and the National Election Watch (NEW) work
towards promoting transparency and accountability in the electoral process.
 They conduct research on candidates' criminal records and financial background, and also campaign for
electoral reforms such as the implementation of paper trails in electronic voting machines.
Transparency and Accountability:
 NGOs like the Centre for Public Interest Litigation (CPIL), the Commonwealth Human Rights Initiative
(CHRI), and the National Campaign for Peoples’ Right to Information (NCPRI) work towards promoting
transparency and accountability in governance.
 They use advocacy and litigation to hold public officials accountable and promote transparency in
government functioning.
Participatory Governance:
 NGOs like the National Centre for Advocacy Studies (NCAS), the Centre for Science and Environment (CSE),
and the Society for Participatory Research in Asia (PRIA) work towards promoting participatory
governance.
 They work with communities to build their capacity to participate in decision-making processes and ensure
that their voices are heard.
What are the challenges faced by NGOs in India:

TSPSC | Group-2 | Batch-3 | Admissions in progress | Call 7013495019 for admissions


 Legal Challenges: NGOs in India also face legal challenges. They have to comply with complex legal
requirements and regulations, which can be time-consuming and expensive. Additionally, NGOs that
engage in advocacy and lobbying can face legal challenges from the government and other stakeholders.
 Capacity Building: NGOs often lack the necessary skills, expertise, and resources to carry out their activities
effectively. Building capacity is essential to ensure that NGOs can operate sustainably and achieve their
objectives
 Limited Access to Information: NGOs also face challenges in accessing information. The government often
withholds information or provides limited access, making it difficult for NGOs to conduct research or
monitor government activities.
Recent Context:
 Surveillance & Clampdown: 'NGOs to be linked to central data hub with unique ID'
 Non-governmental organisations (NGOs) across the country received foreign contributions worth Rs
88,882 crore between 2017-18 and 2021-22.
 This is the period that witnessed a crackdown on NGOs allegedly violating provisions of the Foreign
Contribution (Regulation) Act, with the Act itself being made more stringent.
Conclusion:
In conclusion, overcoming the challenges faced by NGOs in India requires a collaborative effort from different
stakeholders, including the government, corporate partners, donors, and other NGOs. By working together
and adopting innovative approaches, NGOs can overcome the challenges they face and continue to contribute
to development and social justice in India.
10. A. Role of ethics and moral values is significant in bringing good governance. In this context discuss various
ethical and moral values to strengthen governance in India with suitable examples.
Approach:
1. Identify the Keyword “Discuss”.
2. Introduction: Briefly define ethics and moral values
3. Body: Give explanation on how ethics contributed in strengthen governance in India.
4. Conclusion: Give a holistic conclusion.
Model Answer:
Ethics and moral values play a crucial role in strengthening good governance in India. Good governance is
essential for sustainable development, economic growth, and the well-being of citizens.
Here are some ethical and moral values that can contribute to strengthening governance in India, along with
examples of their application:
 Integrity: Integrity refers to the adherence to ethical and moral principles and values, regardless of the
situation or circumstances. In governance, integrity is critical as it promotes transparency, accountability,
and trust.
 For example, the Central Vigilance Commission (CVC) in India plays a vital role in promoting integrity
in governance by ensuring transparency and accountability in government procurement and
contracts.
 Responsibility: Responsibility refers to the sense of obligation towards fulfilling one's duties and
obligations. In governance, responsibility is crucial for ensuring efficient and effective service delivery to
citizens.
 For example, the Swachh Bharat Abhiyan (Clean India Mission) is an initiative launched by the Indian
government to promote responsibility among citizens towards cleanliness and sanitation.
 Fairness: Fairness refers to the equitable and impartial treatment of all individuals, regardless of their
status or background. In governance, fairness is crucial for promoting social justice and equality.

TSPSC | Group-2 | Batch-3 | Admissions in progress | Call 7013495019 for admissions


 For example, the Right to Information (RTI) Act, which allows citizens to access government
information, promotes fairness by ensuring transparency in governance and promoting citizen
participation in decision-making.
 Honesty: Honesty refers to the adherence to truth and sincerity in one's actions and intentions. In
governance, honesty is critical for promoting accountability and transparency.
 For example, the Digital India initiative launched by the Indian government aims to promote honesty
and transparency in government service delivery by reducing corruption and promoting e-governance.
 Respect: Respect refers to the recognition and consideration of the rights, needs, and dignity of all
individuals. In governance, respect is critical for promoting inclusion and diversity.
 For example, the National Rural Livelihoods Mission (NRLM) launched by the Indian government aims
to promote respect for the rights and needs of marginalized communities by empowering them
through skill development and financial assistance.
Ethics and moral values are essential for strengthening governance in India. By promoting integrity,
responsibility, fairness, honesty, and respect, individuals and organizations can create a more transparent,
accountable, and inclusive governance system that benefits all citizens.
10. B. Citizen Charter is an important tool for promoting good governance. Discuss
Approach:
1. Identify the Keyword “Discuss”.
2. Introduction: Briefly define about Citizen Charter
3. Body: Briefly explain about what, why and how of Citizen Charter
4. Conclusion: Give a moderate conclusion.
Model Answer:
Citizen Charter is an essential tool for promoting good governance. It is a document that outlines the
commitments and standards of a government or organization towards providing quality services to its citizens.
The Charter is designed to promote transparency, accountability, and citizen participation in governance.
Objectives
 The goal of the Citizens Charter is to empower citizens in terms of public service delivery.
 The original six principles of the Citizens Charter movement were:
 Quality: Improving service quality.
 Choice: Whenever possible.
 Standard: Specify what to anticipate and how to respond if standards are not reached.
 Value: For the money paid by taxpayers.
 Accountability: Individuals and Organisations are both held accountable.
 Transparency: Rules/Procedures/Scheme/Grievances.
Importance of Citizen Charter
 The Citizen's Charter is an important tool for promoting good governance as it sets measurable standards
for organizations, improves the quality of services, and facilitates monitoring both internally and
externally.
 It also provides clear information about services, reduces costs and delays, and promotes a customer-
centric environment.
 The Charter enhances administrative efficiency, increases citizen participation, reduces corruption
through transparency and accountability, and sets high expectations for organizations, motivating
employees to work diligently.
Limitations and Challenges
 The Charter is not legally binding, which can reduce its effectiveness.
 Citizen responsibilities are not mentioned in the Charter.
 Organizations may not be willing to adhere to their Charters, leading to poor implementation.

TSPSC | Group-2 | Batch-3 | Admissions in progress | Call 7013495019 for admissions


 A one-size-fits-all approach to the Charter may not be appropriate for all organizations or departments.
 The Charter is often not developed through inclusive consultations with stakeholders.
 The needs of the differently-abled and senior citizens are not adequately considered in the Charter
 Lack of awareness and knowledge among the people creates a lack of trust in the Citizen's Charter.
 Inadequate training of staff can lead to poor implementation of the Charter.
 The specified standards are often not quantifiable, making them difficult to measure and achieve.
 The number of accepted charters across the country is limited, and there is a long way to go in achieving
universal acceptance.
 Citizen Charter is crucial to prioritize citizen's needs in any service delivery system. By providing explicit
and visible provisions, it indirectly strengthens the protection of fundamental rights. Adopting successful
models like the Sevottam Model can assist in making Citizen's Charter more efficient and focused on the
citizen. However, there is still much scope for improvement in the implementation of Citizen's Charter
Additional Information:
Suggested Reforms
 The Second ARC concurred that the Citizen’s Charter had become nothing but a set of pious declarations
by ministries and departments. In light of this observation, it recommended some reforms:
 Wider base of consultation while drafting the Citizen’s Charter.
 Orientation of staff towards the salient features of the Charter.
 Devising mechanisms for firm commitments.
 A steady and efficient grievance redressal mechanism.
 Periodic evaluation and evolution of the Charter as needed with time.
Suggestions to improve Citizen Charter implementation are:
 Decentralize the process: The Citizen Charter should be designed as a decentralized effort, with basic
instructions provided by the main office.
 Broad consultation process: A broad consultation process should be used to develop the Citizen Charter,
including consultations within the organization and meaningful communication with civil society.
 Specific commitments: The Citizen Charter should make firm and specific commitments to citizens or
customers in quantifiable terms wherever possible.
 Redressal mechanism: The Charter should provide a redressal mechanism in case of default, explicitly
stating the remedies that the organization is obligated to provide if it fails to meet the stated service
standards.
 Periodic evaluation: The Citizen Charter should be periodically evaluated, preferably by an outside
organization.
 Accountability: Officers should be held accountable for outcomes in cases of failure to follow the Citizen
Charter.
 Involving society: Civil society should be incorporated in the Charter to assist in enhancing its contents.
They should be involved in the process, as well as in educating people about the relevance of the Citizen
Charter's crucial mechanism.
Citizen’s Charter outlines the service commitment of organizations or service providers towards providing
quality, high-standard services, including mechanisms for grievance redressal that promotes ethical
governance and build trustworthiness on the administration.

TSPSC | Group-2 | Batch-3 | Admissions in progress | Call 7013495019 for admissions

You might also like